Download as pdf or txt
Download as pdf or txt
You are on page 1of 59

ForumIAS

Prelims Marathon
June
2018

HISTORY
ECONOMICS
POLITY
SCIENCE AND TECHNOLOGY
GEOGRAPHY AND ENVIRONMENT
PRELIMS MARATHON COMPILATION FOR THE MONTH OF JUNE, 2018

History
Q.1) With reference to Subsidiary Alliance during the company's rule, consider the following statements:
1. Lord Wellesley invented subsidiary alliance system.
2. The Indian state was to surrender its external relation to the care of the company and make no wars
without permission of the Company.
Which of the statements given above is/are correct?
a) 1 only
b) 2 only
c) Both 1 and 2
d) Neither 1 nor 2

Q.2) With reference to the Revolt of 1857, consider the following statements:
1. Sepoys from all of North India participated in the revolt.
2. Lord Canning was the governor general during the revolt.
3. The revolt was mainly feudal in character.
Which of the statements given above is/are correct?
a) 1 only
b) 2 only
c) 2 and 3 only
d) 1, 2 and 3

Q.3) With reference to Raja Ram Mohan Roy, which of the following statements is/are correct?
1. He started the journal ‘Samachar Darpan’.
2. He translated the Vedas and Upanishads into Bengali, Hindi, and English.
Select the correct answer using the codes given below.
a) 1 only
b) 2 only
c) Both 1 and 2
d) Neither 1 nor 2

Q.4) Who among the following was associated with The Fairaizi’s Movement?
a) Chittur Singh
b) Dadu Miyan
c) Sidhu and Kanhu
d) Velu Thampi

Q.5) With reference to uprisings during colonial rule, consider the following events.
1. Chuar and Ho Revolt
2. Munda Rebellion
3. Sepoy Mutiny
What is the correct chronological sequence of the above events?
Created with by ForumIAS.com – The Knowledge Network for Civil Services.
Visit http//forumias.com New! http://forumiasacademy – Prelims & Mains Test Series

ForumIAS
PRELIMS MARATHON COMPILATION FOR THE MONTH OF JUNE, 2018

a) 1 – 2 – 3
b) 2 – 1 - 3
c) 3 – 2 - 1
d) 1 – 3 – 2

Q.6) Consider the following statements regarding Carnatic wars


1. The Carnatic wars were fought between British and French.
2. It was influenced by Anglo French war in Europe.
Which of the above given statements is/are incorrect?
a) 1 only
b) 2 only
c) Both 1 and 2
d) Neither 1 nor 2

Q.7) Consider the following statements about Lord Cornwallis.


1. He was the first Governor General of Bengal.
2. He Introduced reform based on separation of power.
3. A new post of session judge was created by him to preside over the district civil court.
Which of the statements given above is/are correct?
a) 2 and 3 only
b) 1 and 3 only
c) 2 only
d) 1, 2 and 3

Q.8) Consider the following statements with regard to Ranjit Singh:


1. He fought against British in the first Anglo Sikh war 1845.
2. He struck coins in the name of Guru Nanak.
3. He employed Europeans in the service of state.
Which of the statement given above is/are correct?
a) 1 only
b) 1 and 3 only
c) 2 and 3 only
d) 1, 2 and 3

Q.9) With reference to the impact of the British Rule on Indian agriculture, consider the following
statements:
1. British policies led to the recurring famines.
2. British policies increased the percentage of population dependent on agriculture.
3. Commercialisation of agriculture led to the exploitation of cultivators by moneylenders and merchants.
Which of the statements given above is/are correct?
a) 1 only
b) 2 and 3 only
c) 1 and 3 only
d) 1, 2 and 3

Created with by ForumIAS.com – The Knowledge Network for Civil Services.


Visit http//forumias.com New! http://forumiasacademy – Prelims & Mains Test Series

ForumIAS
PRELIMS MARATHON COMPILATION FOR THE MONTH OF JUNE, 2018

Q.10) Which of the following pairs is/are incorrectly matched?


1. Hyderabad: Hyder ali
2. Bengal: Aliwardi Khan
3. Awadh: Siraj Ud daula
Select the correct Answer using the codes given below.
a) 1 and 3 only
b) 1 and 2 only
c) 2 only
d) 1, 2 and 3

Q.11) Consider the following statements with regard to the Third Battle of Panipat:
1. It was fought between Nadir shah and Maratha.
2. Maratha conquest of Punjab was one of the reason for the war.
3. Most of the Muslim ruler of North India fought against Maratha.
Which of the statements given above is/arecorrect?
a) 1 only
b) 2 and 3 only
c) 1 and 3 only
d) 1, 2 and 3

Q.12) Consider the following statements:


1. ‘Khuda-kasht’ were peasants with occupancy rights.
2. 'Dadani' was a System where a merchant procured goods by paying advance money to primary
producer
Which of the statements given above is/are correct?
a) 1 only
b) 2 only
c) Both 1 and 2
d) Neither 1 nor 2

Q.13) Who were 'Gomustah' during the British rule?


a) British Official working for Indian states
b) Sikh band of warriors
c) Village headmen
d) Indian Agent of East India Company

Q.14) Consider the following statements:


1. Racial Bitterness against Indian decreased.
2. Tight European control over the civil and military administration
3. Development of representative institutions in India
Which of the above is/are the consequences of the Revolt of 1857?
a) 1 only
b) 1 and 2 only
c) 2 and 3 only
d) 1,2 and 3
Created with by ForumIAS.com – The Knowledge Network for Civil Services.
Visit http//forumias.com New! http://forumiasacademy – Prelims & Mains Test Series

ForumIAS
PRELIMS MARATHON COMPILATION FOR THE MONTH OF JUNE, 2018

Q.15.Arrange the following in the order of occurrence:


1. The August Offer
2. Wavell’s plan
3. Cripp’s Mission
4. Quit India Movement
Select the correct answer using the code given below:
a) 1-3-4-2
b) 3-1-4-2
c) 1-3-2-4
d) 3-1-2-4

Q.16) Arrange the following in the order of the year of occurrence:


1. Formation of Swarajist party
2. Montagu-Chelmsford reforms
3. All-India Forward Bloc
4. Government of India Act-1935
Select the correct answer using the code given below:
a) 1-2-3-4
b) 2-1-4-3
c) 2-1-3-4
d) 1-2-4-3

Q.17 The Sadler Commission was appointed to review


a) Account the ways & means of improving education facility
b) Importance of training of teachers in Native language
c) Administrational & educational problems of Calcutta University
d) Development schemes of education

Q.18). Who among the following kingdom sent an embassy to roman emperor Augustus?
a) Cholas
b) Cheras
c) Pandyas
d) Satavahanas

Q.19) Which Major Rock Edict (MRE) talk about replacement of berighosh by dhammagosh as a policy of
Ashoka?
a) MRE VII
b) MRE IX
c) MRE XI
d) MRE XIII

Q.20 ‘Grama Bhojaka’ refers to


a) Village treasures
b) Village headman
Created with by ForumIAS.com – The Knowledge Network for Civil Services.
Visit http//forumias.com New! http://forumiasacademy – Prelims & Mains Test Series

ForumIAS
PRELIMS MARATHON COMPILATION FOR THE MONTH OF JUNE, 2018

c) Village priest
d) None

Q.21 Which of the following statements is/are correct with reference to ‘Nehru Report’?
1. Jawahar Lal Nehru was member of this.
2. It recommended the complete Independence and joint electorates for reservation of seats for minorities.
3. It recommended the ‘Fundamental Rights’ to all Indians.
Select the correct option using code given below
a) 1 and 2 only
b) 1 and 3 only
c) 3 only
d) 1,2 and 3

Q.22) The Congress rejected the August Offer 1940 because


a) India did not want to participate in World War II.
b) Number of Indians in Viceroy’s Executive Council was decreased.
c) Demand for provincial national government was rejected by British
d) All the above

Q.23) Which of the following statements is/are correct about the All Indian Women’s Conference?
1. It was started through the effort of Mrs. Margaret Cousins.
2. It had played crucial role to pass the Sarda Act.
Select the correct answer using code given below:
a) 1 only
b) 2 only
c) Both 1 and 2
d) Neither 1 and 2

Q.24) Which of the following school of Indian philosophy believed that priest is required for attaining
one’s salvation performing with their all rituals?
a) YOGA
b) SANKHYA
c) MIMANSA
d) NYAYA

Q.25) The teachings of buddha were highly practised at various learning centres in India during the reign
of various rulers who patronisedbuddhism. The following universities were built during the reign of
PALA rulers except—
a) Vikramashila university
b) Odantpuri university
c) Jaggadala university
d) Vallabhi

Created with by ForumIAS.com – The Knowledge Network for Civil Services.


Visit http//forumias.com New! http://forumiasacademy – Prelims & Mains Test Series

ForumIAS
PRELIMS MARATHON COMPILATION FOR THE MONTH OF JUNE, 2018

Q.26) With the reference to the Sallekhana, consider the following statements:
1. It is the last vow prescribed by the Jain ethical code of conduct.
2. The duration of the practice could be upto eight to ten days
3. In 2015, the Supreme Court stayed the decision of the Rajasthan High Court and put a ban on
sallekhana.
Which of the statements given above is/are incorrect?
a) 2 and 3
b) 2 Only
c) 3 Only
d) 1, 2 and 3

Q.27) Which of the following are correct regarding Sulva sutras?


1. Earliest book on Mathematics
2. Had mention of geometry
3. Mentioned zero
4. Is written by Bandhyan
Select the correct answer using code given below:
a) 2 and 3
b) 3 only
c) 1, 2 and 4
d) 2, 3 and 4

Q.28) Who among the following have written the metrical legendary and historical chronicle of the north-
western Indian subcontinent, Rajatarangini?
a) Megasthenese
b) Kalhana
c) Al-Biruni
d) Herodotus

Answer Key

Qs. Ans. Qs. Ans. Qs. Ans.


1 B 11 B 21 C
2 C 12 C 22 C
3 B 13 D 23 C
4 B 14 C 24 C
5 D 15 A 25 D
6 D 16 B 26 A
7 A 17 C 27 C
8 C 18 C 28 B
9 D 19 D
10 A 20 B
Created with by ForumIAS.com – The Knowledge Network for Civil Services.
Visit http//forumias.com New! http://forumiasacademy – Prelims & Mains Test Series

ForumIAS
PRELIMS MARATHON COMPILATION FOR THE MONTH OF JUNE, 2018

Explanaiton

1. Terms of subsidiary alliance system:


● An Indian ruler entering into a subsidiary alliance with the British would accept British forces
within his territory and to pay for their maintenance.
1. The ruler would accept a British Resident in his state.
2. The ruler who entered into a subsidiary alliance would not join any alliance with any other
power or declare war against any power without the permission of the British.
3. The ruler would dismiss any Europeans other than the British and avoid employing new ones.
4. The ruler would let the British to resolve any conflict with any other state.
5. The ruler would have his state be protected by the Company from external dangers and internal
disorders.
6. If the rulers failed to make the payments that were required by the alliance, part of their territory
would be taken away as a penalty.

2. Punjab and Kashmir supported the British. Not whole of North India participated in the revolt. Hence,
statement 1 is incorrect.
Charles Canning, was the Governor-General of India during the rebellion of 1857. Hence, statement 2 is
correct.
Jawaharlal Nehru in his book Discovery of India maintains that the Revolt of 1857 was essentially ‘a feudal
uprising though there were some nationalistic elements in it’. This was earlier professed by Moderates. Hence,
statement 3 is correct.

3. Raja Ram Mohun Roy


● He was a polyglot who had mastered various languages such as Sanskrit, Persian, Arabic,
English, Bengali and Hindi
● He was the founder of the Brahma Samaj, which played a major role in reforming and
modernising the Indian society
● He formed the Atmiya Sabha as a philosophical discussion circle to debate monotheistic Hindu
Vedantism and similar subjects
● He was against idol worship and orthodox Hindu rituals
● He denounced the caste system
● He started a Bengalee journal ‘Sambad Kaumudi’
Samachar Darpan was first Bengalee newspaper published 1818 by Baptist Missionary Society. Hence,
statement 1 is incorrect.

4. The Faraizi Movement was founded in 1818 by Haji Shariatullah to give up un-Islamic practices and
act upon their duties as Muslims. After the death of Haji Shariatullah, his son, Dadu Miyan, led the

Created with by ForumIAS.com – The Knowledge Network for Civil Services.


Visit http//forumias.com New! http://forumiasacademy – Prelims & Mains Test Series

ForumIAS
PRELIMS MARATHON COMPILATION FOR THE MONTH OF JUNE, 2018

movement to a more agrarian character. He organised the oppressed peasantry against the oppressive
landlords.
Chittur Singh: Leader of Ramosi uprising.
Sidhu and Kanhu: Leaders of Santhal rebellion. Velu Thampi: Led Travancore rebellion against the
British East India Company

5. 1. Chuar and Ho Revolt - 1767-72


2. Munda Rebellion - 1899-1900
3. Sepoy Mutiny - 1857

6. In the 18th century, the coastal Carnatic region was a dependency of Hyderabad.
Three Carnatic Wars were fought between 1746 and 1763 between British and French. Hence, statement 1
is correct.
It began with Austrian War of Succession and ended with Seven Year War that took place in Europe.
Hence, statement 2 is correct.
After the war French company was pushed to a corner and was confined primarily to Pondicherry while
The East India company's dominance eventually led to control by the British Company over most of
India.

7. Lord Warren Hasting was first governor general of Bengal. Hence, statement 1 is incorrect.
The Cornwallis Code of 1793:
A comprehensive body of rules dealing with every department of the state was drawn up. Clear division
between the administrative and commercial services was made. Hence, statement 2 is correct.
The reform of Cornwallis in the field of justice consisted chiefly in removing the Indian judges and
replacing them by Europeans with defined powers. The Collector was invested with certain powers of
Criminal justice which he exercises even today.
At the head of each district he appointed a session’s judge. Hence, statement 3 is correct.
Other field where Cornwallis Code initiated reforms were: Police, Land Revenue, Criminal Law,
Suppression of Bribery, Europeanization of Administration machinery.
Source: Modern Indian History B L Grover

8. Ranjit Singh maintained a friendly relation with British. He died in 1839. It was in 1845 that First Anglo
Sikh War was fought. Hence, statement 1 is incorrect.
Maharaja Ranjit Singh organized the Sikh Empire, the first coins that were minted under him were
dedicated to the first guru, Guru Nanak Dev Ji. The coins minted under him always bore the inscription
of leaf of Banyan, a symbol of his empire. During his rule, the coins were minted at eight locations,
Amritsar, Anandgarh, Lahore, Peshawar, Multan, Jammu and Kashmir. Hence, statement 2 is correct.
Ranjit Singh’s army included a few Europeans like Jean-François Allard. Through employment contracts,
he inculcated strict discipline like that they should not eat beef, not smoke, not cut their hair, and asked
them to marry and settle down with Indian women. Hence, statement 3 is correct.

Created with by ForumIAS.com – The Knowledge Network for Civil Services.


Visit http//forumias.com New! http://forumiasacademy – Prelims & Mains Test Series

ForumIAS
PRELIMS MARATHON COMPILATION FOR THE MONTH OF JUNE, 2018

9. The recurring famines of 19th century were the inevitable consequences of the British policies and
exposed the real character of the British administration for Indian peasantry. In the aftermath of great
famine of 1866 Government appointed George Campbell Commission to investigate the causes of famine
and to recommend measures to prevent recurrences in future.
The Committee held government system responsible for creating the famine like conditions and
suggested that the government during famine times must organize the relief measures. Hence, statement
1 is correct.
British conquest led to the de-industrialisation of the country and increased dependence of the people on
agriculture. Hence, statement 2 is correct.
The policy of commercialization of agriculture by the British encouraged market oriented production of
cash crops. Indian peasants were forced to grow these cash crops that spoiled the fertility of the land and
no other crop could be grown on it. The growth of minimum of subsistence crops led to the deterioration
and impoverishment of the Indian agriculture and the cultivators. The extraction of exorbitant rents by
the government oppressed the peasants heavily. In order to meet the high demand of revenue, the
peasants perpetually remained indebted to the local money-lenders. Hence, statement 3 is correct.

10. 1. Mysore: Hyder ali


2. Hyderabad: Nizam Ul mulk
3. Bengal: Aliwardi Khan, Siraj Ud Daulah was the last independent Nawab of Bengal
4. Awadh: Saadat khan

11. In 1761 Third battle of Panipat was fought betwen Ahmad Shah Abdali and Maratha. Hence,
statement 1 is incorrect.
The Marathas had gained control of considerable part of India in the intervening period (1707–1757). In
1758 they occupied Delhi, captured Lahore and drove out Timur Shah Durrani, the son and viceroy of the
Afghan ruler, Ahmad Shah Abdali. This was a high-water mark of the Maratha expansion and ultimately
became the reason for Third battle of Panipat. Hence, statement 2 is correct.
Ahmad Shah Abdali, was supported by two Indian allies—the Rohilla Afghans of the Doab, and Shuja-
ud-Daula, the Nawab of Awadh. Hence, statement 3 is correct.

12. During Mughal rule Khud-kasht (riyayati) were those residential peasants living in their own village,
having ownership of lands and implements, paying the land-revenue at a concessional rate, formed the
governing body of the village community. Hence, statement 1 is correct.
During Mughal Rule, the merchants in order to ensure supply, advanced loan to weavers to enable them
to buy yarn. This system was known as Dadani. Hence, statement 2 iscorrect.

13. Gomustah- Indian Agent of East India Company


Source: Plassey to Partition: Sekhara Bandopadhyaya: Glossary

14. In the aftermath of Revolt of 1857:


Created with by ForumIAS.com – The Knowledge Network for Civil Services.
Visit http//forumias.com New! http://forumiasacademy – Prelims & Mains Test Series

ForumIAS
PRELIMS MARATHON COMPILATION FOR THE MONTH OF JUNE, 2018

1. The proportion of Indian soldiers in the army was reduced.


2. It was considered that Muslims were majorily responsible for the rebellion. Hence their land and
property was confiscated.
3. In the post-Revolt period, to maintain supremacy in India, British followed the policy of communal
disharmony.
4. The British adopted a policy of opposing the educated middle class and supporting the landlords and
the native princes
It can be concluded from the above points that racial bitterness actually increased. Hence, statement 1 is
incorrect.
Control of India was snatched from East India Company and passed on to the Crown.
The supreme executive and legislative authority in India henceforth came to be known as the Viceroy and
Lord Canning so far known as the Governor General of Bengal also became first Viceroy of India. Also, a
Post of ‘Secretary of State for India’ was created who was responsible to British Parliament. Hence,
statement 2 is correct.
Britishers came to know that it is hard to rule India without their support. Government of India act 1858,
Indian councils acts of 1861, 1892, 1909 etc. were passed in British parliament. With this decentralised
governance started in India. Demand for development of representative institutions also happened from
Indian side with initiation of many associations finally culminating into formation of Congress in 1885.
Hence, statement 3 is correct.

15. (i) The August Offer-1940


(ii) Wavell’s plan-1945
(iii) Cripp’s Mission-1942(March)
(iv) Quit India Movement 1942 (august)

16. (i) Formation of Swarajist party-1924


(ii) Montagu-Chelmsford reforms-1919
(iii) All-India Forward Bloc -1939
(iv) Government of India act-1935

17. In 1917 the government appointed the Sadler Commission to inquire into the “conditions and
prospects of the University of Calcutta,” an inquiry that was in reality nationwide in scope. Covering a
wide field, the commission recommended the formation of a board with full powers to control secondary
and intermediate education)

18. The Pandyan country was wealthy and prosperous as they profited from trade with Roman Empire
and sent embassies.

19. MRE VII - tolerance among all sects


MRE IX - avoiding expensive and meaningless ceremonies and rituals.
MRE XI - charity, kinship, dhamma
Created with by ForumIAS.com – The Knowledge Network for Civil Services.
Visit http//forumias.com New! http://forumiasacademy – Prelims & Mains Test Series

ForumIAS
PRELIMS MARATHON COMPILATION FOR THE MONTH OF JUNE, 2018

MRE XIII - change of heart after Kalinga war. berighos (sound of war drums) by dhammagosha (sound of
peace).

20. The village headmen were known as grama bhojaka. Most of the time the position was held by the same
family. Grama bhojaka carried out multifarious functions in the village. He was responsible for
maintaining the law and order and settling the disputes among the people.

21. The Nehru Report was headed by Motilal Nehruand Jawahar Lal Nehru was not the member of the
Report. The report recommended that a declaration of Rights should be inserted in constitution assuring
the fullest liberty of conscience and religion. The followings were the recommendation of the Report
1. India should be given the status of dominion.
2. Federal form of government with residuary powers should be vested in the centre.
3. Bicameral legislature
4. No separate electorate for any community, however reservation for Muslims in those provinces where
Muslim population was at least 10%.
5. Separation of Sindh from Bombay, full provincial status to NWFP.
6. Hindi should be made official language of India. Complete independence was declared during Lahore
session presided by Jawahar Lal Nehru

22. When the Congress ministries in the Provinces resigned, the British wanted to get support of the
Congress for war. This was responded by Lord Linlithgow in the sort of a proposal which is called
August Offer. The Congress did not approve the August Offer. Jawahar Lal Nehru said that the whole
idea was “dead and doornail”.

23. The All India Women’s Conference (AIWC) is a non-governmetalorganisation (NGO) based in Delhi.
It was founded in 1927 by margaretCousins ”as an organisation dedicated to upliftment and betterment
of women and children”. It had played a crucial role to pass sarda act-raising the marriage age of girl.

24. The aim of Mimamsa is to give rules for the interpretation of the Vedas. The goal of Mimamsa is to
provide enlightenment on dharma. The set of ritual obligations and prerogatives, if properly performed,
maintains the harmony of the world. Since dharma cannot be known through either perception or
reasoning, one must depend on revelation in the Vedas, which are considered eternal and infallible.

25. The Maitraka kings who ruled Western India constructed a monastery at Vallabhi their capital. While
Nalanda was the centre for Mahayana Buddhism, Vallabhi achieved fame as the centre for Hinayana
Buddhism. The Maitraka kings spent lavishly to maintain the university. They gave every encouragement
and assistance to Buddhist studies at this institution. In the 7th century Vallabhi was as prosperous and
famous as Nalanda.Odantapuri was a Buddhist Mahavihara in what is now Bihar, India. It was
established by the Pala Emperor Gopala I in the 8th century. JagaddalaMahavihara (fl. late 11th century -
mid-12th century) was a Buddhist monastery and seat of learning in Varendra, a geographical unit in
present north Bengal in Bangladesh.[1] It was founded by the later kings of the Pāla dynasty, probably
Ramapala (c. 1077-1120), most likely at a site near the present village of Jagdal in DhamoirhatUpazila in
the north-west Bangladesh on the border with India, near Paharapur.

Created with by ForumIAS.com – The Knowledge Network for Civil Services.


Visit http//forumias.com New! http://forumiasacademy – Prelims & Mains Test Series

ForumIAS
PRELIMS MARATHON COMPILATION FOR THE MONTH OF JUNE, 2018

26. Statement 1 is true It is the last vow prescribed by the Jain ethical code of conduct.The vow of
sallekhana is observed by the Jain ascetics and lay votaries at the end of their life by gradually reducing
the intake of food and liquids.Sallekhana is allowed when normal life according to religion is not possible
due to old age, incurable disease or when a person is nearing his end.It is a highly respected practice
among the members of the Jain community.
Statement 2 is false The duration of the practice could be up to twelve years or more.
Statement 3 is false Thousands of Jain community members took out a silent march to protest the recent
Rajasthan High Court order banning the practice of Santhara or Sallekhana.The Rajasthan high court had
held that the practice of Santhara or Sallekhana is a form of suicide and is illegal, eliciting protests by the
community across the country.The Supreme Court stayed the decision of the Rajasthan High Court and
lifted the ban on Sallekhana

27. The rules governing the use of zero appeared for the first time in
Brahmagupta'sBrahmasputhaSiddhanta

28. Kalhana'sRajatarangini is one of the works of history which is indeed a solitary example of its kind
written between 1148 and 1149. It enjoys great respect among the historians for its approach and
historical content. Kalhana’s work of Rajatarangini consists of 7826 verses, which are divided into eight
books called Tarangas ("waves").

Created with by ForumIAS.com – The Knowledge Network for Civil Services.


Visit http//forumias.com New! http://forumiasacademy – Prelims & Mains Test Series

ForumIAS
PRELIMS MARATHON COMPILATION FOR THE MONTH OF JUNE, 2018

Economy

Q.1) Consider the following:


1. Fringe benefit tax
2. Interest tax
3. Securities transaction tax
Which of the above is/ are Direct Tax/ Taxes?
a) 1 only
b) 1 and 3 only
c) 2 and 3 only
d) 1, 2 and 3

Q.2) with reference to India, consider the following :


1. Nationalization of banks
2. Formation of regional Rural banks
3. Adoption of villages by bank branches
Which of the above can be considered as steps taken to achieve the “financial inclusion”
In India?
a) 1 and 2 only
b) 2 and 3 only
c) 3 only
d) 1, 2 and 3

Q.3) India has experienced persistent, and high food inflation in the recent past, what could be the
reasons ?
1. Due to gradual switch over the cultivation of commercial crops, the area under the cultivation of food
grains has steadily decreased in the last five years by about 30%
2. As a consequence of increasing incomes, the consumption patterns of the people have under gone a
significant change.
3. the food supply chain has structural constraints.
Which of the statements given above the correct ?
a) 1 and 2 only
b) 2 and 3 only
c) 1 and 3 only
d) 1, 2 and 3

Q.4) Consider the following actions which the government can take:
1. Devaluing the domestic currency.
2. Reduction in the export subsidy
Created with by ForumIAS.com – The Knowledge Network for Civil Services.
Visit http//forumias.com New! http://forumiasacademy – Prelims & Mains Test Series

ForumIAS
PRELIMS MARATHON COMPILATION FOR THE MONTH OF JUNE, 2018

3. Adopting suitable policies which attract greater FDI and more funds from FIIs.
Which of the above action / actions can help in reducing the current account deficit?
a) 1 and 2
b) 2 and 3
c) 3 only
d) 1 and 3

Q.5) A “closed economy” is an economy in which


a) The money supply is fully controlled
b) deficit financing takes place
c) only exports takes place
d) Neither exports nor imports take place

Q.6) Kar Marx explained the process of class struggle with the help of which one of the following theories
?
a) Empirical liberalism
b) Existentialism
c) Darwin’s theory of evolution
d) Dialectical materialism

Q. 7) Economic Growth is usually coupled with


a) Deflation
b) Inflation
c) stagflation
d) Hyperinflation

Q.8) The lowering of bank Rate by the Reserve Bank of India leads to
a) more liquidity in the market
b) less liquidity in the market
c) No change in the Liquidity in the market
d) Mobilization of more deposits by commercial-banks

Q.9) India is regarded as a country with “ Demographic Dividend” this is due to


a) Its high population in the age group below 15 years
b) its high population in the age group of 15 to 64 years
c) Its high population in the age group above 65 years
d) Its high total population

Created with by ForumIAS.com – The Knowledge Network for Civil Services.


Visit http//forumias.com New! http://forumiasacademy – Prelims & Mains Test Series

ForumIAS
PRELIMS MARATHON COMPILATION FOR THE MONTH OF JUNE, 2018

Q.10) Which of the following can aid in furthering the Government’s objective of inclusive growth?
1. promoting Self Help Groups
2. promoting micro, small and medium enterprises
3. implementing the Right to Education Act.
Select the correct answer using the codes given below:
a) 1 only
b) 1 and 2 only
c) 2 and 3 only
d) 1, 2 and 3

Q.11) which of the following is / are included in Macro Vulnerability Index:


1. Inflation rate
2. Foreign direct investment.
3. Current Account deficit.
4. Fiscal deficit.
Choose the correct from the given option:
A) 1, 2 and 3
B) 1, 3 and 4
C) 3 Only
D) 1, 2 and 4

Q.12) Consider the following statement with reference to Reserve-Replacement Ratio:


A) It is ratio used to judge the operating performance of an oil and gas exploration and production
company
B) It certain percentage of the total bank deposits has to be kept in the current account with RBI
C) The amount of money which remains blocked for statutory reasons and is not available for investment
D) It is reserve money used by Angel investor to overcome the investment burden.

Q.13) Consider the following Statement with reference to Liquidity adjustment facility:
1) LAF is a facility extended by the Reserve Bank of India to the scheduled commercial banks including
RRBs
2)LAF enables liquidity management on a day to day basis.
3)The introduction of Liquidity adjustment facility in India was on the basis of the recommendations of
Narasimham committee.
Choose the correct from the given option:
A) 1 Only
B) 2 and 3
C) 3 Only
D) 1, 2 and 3

Q.14) Consider the following statements about the Cryptocurrency:


1. Cryptocurrency is a digital currency which makes use of encryption technique.
Created with by ForumIAS.com – The Knowledge Network for Civil Services.
Visit http//forumias.com New! http://forumiasacademy – Prelims & Mains Test Series

ForumIAS
PRELIMS MARATHON COMPILATION FOR THE MONTH OF JUNE, 2018

2. Cryptocurrency is operating under the central bank of country concern.


Choose the correct from the given option:
a) 1 Only
b) 2 Only
c) Both 1 and 2
d) Neither 1 nor 2

Q.15) Consider the following statements about Cash Reserve Ratio:


1. It is the ratio of deposits which banks have to keep with RBI .
2. The increase in CRR will lead to increase in lending capacity of banks.
3. Banks get a fixed interest on CRR from RBI
Which of the above statements are incorrect?
a) 1 and 2
b) 2 and 3
c) 1 and 3
d) None of the above

Q.16) The Money Supply in the economy affects which of the following macroeconomic variables:
(a) Rate of Interest
(b) Price level
(c) Output
(d) All of the above

Q.17) With reference to Financial Resolution and Deposit Insurance bill 2017, consider the following
statements:
1.It seeks to establish a Resolution Corporation to replace the existing Deposit Insurance and Credit
Guarantee Corporation(DICGC)
2. Resolution Corporation will monitor banks only and anticipate their risk of failure
Which of the statements given above is/are correct?
a) 1 only
b) 2 only
c) Both 1 and 2
d) Neither 1 nor 2

Q.18) Which of the following will be included in a country's national income?


1. Expenditure on purchase of an old house.
2. Brokerage on sale of shares.
3. Meals given to the beggars.
4. Commission on the sale on second hand car.
Select the correct answer using the codes given below.
a) 1, 2 and 3 only
b) 1, 3 and 4 only
Created with by ForumIAS.com – The Knowledge Network for Civil Services.
Visit http//forumias.com New! http://forumiasacademy – Prelims & Mains Test Series

ForumIAS
PRELIMS MARATHON COMPILATION FOR THE MONTH OF JUNE, 2018

c) 2 and 4 only
d) 1, 2 and 4 only

Q.19) An engineer left his job and started preparing for UPSC exams. This kind of unemployment is
called
a) Frictional
b) Structural
c) Cyclical
d) None

Q.20) Consider the following statements about the purchasing power parity
1.If the real exchange rate is equal to zero, currencies are at purchasing power parity.
2.If the real exchange rises above one, this means that goods abroad have become more expensive than
goods at home.
3.The real exchange rate is taken as a measure of a country’s international competitiveness.
Which of the statements given above is/are correct?
a) 1 and 3 only
b) 2 and 3 only
c) 1, 2 and 3
d) 2 only

Q.21) National Pharmaceutical Pricing Authority can cap the prices of which of the following?
1. Drugs
2. Medical devices
3. Surgical Procedures
Select the correct answer using the code given below.
a) 1 and 2 only
b) 2 and 3 only
c) 1 and 3 only
d) 1, 2 and 3

Q.22) Consider the following instruments of money transaction


1. currency notes and coins
2. savings account deposits
3. current account deposits
4. Fixed deposits
Which of the above given instruments is/are considered as demand deposits
a) 1, 2, 3 and 4
b) 1 only
c) 1, 2 and 3 only
d) 2, 3 and 4 only

Created with by ForumIAS.com – The Knowledge Network for Civil Services.


Visit http//forumias.com New! http://forumiasacademy – Prelims & Mains Test Series

ForumIAS
PRELIMS MARATHON COMPILATION FOR THE MONTH OF JUNE, 2018

Q.23) International Organization for Migration (IOM) releaseWorld Migration Report 2018. Consider the
following statement regarding this.
1. Indian diaspora have been found to be world’s largest.
2. USA tops the list of destination countries of global diaspora.
Which of the above statement(s) is/are correct?
a) 1 Only
b) 2 Only
c) Both 1 & 2
d) Neither 1 nor 2

Q.24) Consider the following statements about Pradhan Mantri Fasal Bima Yojana
1. It is compulsory for farmers availing crop loans for notified crops in notified areas and voluntary for
non-loanee farmers.
2. Post Harvest losses is not covered covered.
Which of the above statement(s) is/are correct?
a) 1 Only
b) 2 Only
c) Both 1 & 2
d) Neither 1 nor 2

Q.25) Consider the following statements about Paramparagat Krishi Vikas Yojana
1. It is to promote organic farming
2. There is no liability for farmers for expenditure on certification.
Which of the above statement(s) is/are correct?
a) 1 Only
b) 2 Only
c) Both 1 & 2
d) Neither 1 nor 2

Q.26) Consider the following statements about Electronic National Agriculture Market (e-NAM)
1. It networks the existing APMC mandis to create a unified national market for agricultural commodities
2. NAM is a virtual market and it is not connected to a physical market
Which of the above statement(s) is/are correct?
a) 1 Only
b) 2 Only
c) Both 1 & 2
d) Neither 1 nor 2

Q.27) Consider the following statements recently launched bharat net


1. To provide network infrastructure with affordable broadband connectivity on a non-discriminatory
basis to all households in the country.
2. It is in partnership with States and the Private Sector.
Created with by ForumIAS.com – The Knowledge Network for Civil Services.
Visit http//forumias.com New! http://forumiasacademy – Prelims & Mains Test Series

ForumIAS
PRELIMS MARATHON COMPILATION FOR THE MONTH OF JUNE, 2018

Which of the above statement(s) is/are correct?


a) 1 Only
b) 2 Only
c) Both 1 & 2
d) Neither 1 nor 2

Answer Key

Qs. Ans. Qs. Ans. Qs. Ans.


1 D 11 B 21 A
2 D 12 A 22 C
3 B 13 B 23 C
4 D 14 A 24 A
5 D 15 B 25 C
6 D 16 D 26 A
7 B 17 A 27 C
8 A 18 C
9 B 19 A
10 D 20 B

Explanation

1. Direct tax is a tax directly payed by an individual or organistion to the imposing entity. Example of
direct tax are-
1. Corporation
2. Tax
3. Securities transaction
4. Banking cash transaction
5. Fringe benefits tax
6. Wealth tax
7. Interest tax.
Fringe benefit tax is applied to employees, for the expense made for providing services to the employees,
which cannot otherwise accounted individually
Interest tax is direct tax imposed on interest accrued in specific cases. However, it was abolished im
March 2000
Securities transaction tax is a tax applied in the transaction of stock market, during a transaction. It’s
applied both on buyers as well as seller.
Indirect taxes are levied on entity such as seller and paid by another such as tax paid by the buyer in a
retail setting. Examples of Indirect tax – 1. Union Excise duty 2. Custom duty 3. Service tax
Created with by ForumIAS.com – The Knowledge Network for Civil Services.
Visit http//forumias.com New! http://forumiasacademy – Prelims & Mains Test Series

ForumIAS
PRELIMS MARATHON COMPILATION FOR THE MONTH OF JUNE, 2018

2. Financial inclusion or inclusive financing is the delivery of financial services at affordable costs to
section of disadvantage and low- income segment of society , in contrast to financial exclusion where
those service are not available or affordable.
In this particular question all the steps are targeted financial inclusion in one way or the other
nationalization targeted expansion of bank branches in rural areas and also a control on concentration of
economic powers in hands of few. RRBs were planned to cater the needs of rural areas.

3. Food price inflation has remained persistently elevated for over a year now, reflecting in a part the
structural demand supply mismatches in a several commodities as well as the changing consumption
patterns.

4. Current account deficit means the excess of imports over exports. The current Account Deficit can be
reduced by adopting suitable policies favouring FDI and FII. It is domestic currency is devalued, export
would become cheaper which will create additional demand for product in the world market.
Any reduction in the export subsidy is dangerous as the countries goods will become costlier and exports
would fall

5. An economy in which no activity is conducted with outside economics. A closed economy is self-
sufficient , meaning that no imports are brought in and no exportsare sent out. The goal is to provide
consumers with everything that they need from within the economy’s borders. A closed economy is the
opposite of an open economy, in which a country will conduct trade with outside regions.

6. Dialectical materialism is a philosophy of science and nature, based on the writings of Karl Marx and
Friedrich Engels, and developed largely in Russia and the Soviet Union. The sketch of dialectical
materialism given by Karl Marx explains that Dialectical principle is of universal application and all the
development is struggle of opposites” or “ conflict of opposities”

7. The idea that an increased in economic growth reduces inflation. Here’s why Inflation , as the old
saying goes, is caused by too much money “ chasing” too few goods. Just as means higher prices , fewer
goods also means higher price the connection between the level of production and level of prices also
holds for the rate of change and production (that is, the rate of economic growth) and the rate of change
of prices(that is, the inflation rate). During deflation there is no demand and no growth.

8. Bank rate is the rate interest which a central bank (RBI) charges on the loans and advanced to a
commercial bank. It also referred to as the discount Rate.
If RBI wants to increase the liquidity and money supply in the market , it will be decrease the bank rate
and if it wants to reduce the liquidity and money supply in the system, It will increase the bank rate.

Created with by ForumIAS.com – The Knowledge Network for Civil Services.


Visit http//forumias.com New! http://forumiasacademy – Prelims & Mains Test Series

ForumIAS
PRELIMS MARATHON COMPILATION FOR THE MONTH OF JUNE, 2018

9. The demographic dividened is a rise in the rate of economic growth due to a rising share of working
class people in a population. India’s demographic dividend is its working age (15-34years) and as a result
its economy has the potential to grow more quickly than that of many other countries, including china.

10. Inclusive growth is a concept which advances equitable opportunities for economic participants
during the process of economic growth with benefits incurred by every section of society all the
statements contribute to inclusive growth.

11. Macro Vulnerability Index include Inflation, current account deficit and fiscal deficit. The Index value
can be compared across countries for different time periods to gauge their relative vulnerability.
MVI is determined by various structural conditions which expose an economy to financial shocks.
Macro economic vulnerability are the credibility of Government policies and Economic robustness.

12. The reserve-replacement ratio is a metric used by investors to judge the operating performance of an
oil and gas exploration and production company. The reserve-replacement ratio measures the amount of
proved reserves added to a company's reserve base during the year relative to the amount of oil and gas
produced

13. LAF is a facility extended by the Reserve Bank of India to the scheduled commercial banks (excluding
RRBs) and primary dealers to avail of liquidity in case of requirement or park excess funds with the RBI
in case of excess liquidity on an overnight basis against the collateral of Government securities including
State Government securities.
The introduction of Liquidity adjustment facility in India was on the basis of the recommendations of
Narsimham committee on banking sector reforms.

14. Cryptocurrency is a digital currency which makes use of encryption techniques to regulate the
generation of units of currency and to verify the transfer of funds.
Cryptocurrency is operating independently of a central bank.
This is a math-based, decentralised convertible virtual currency that is protected by cryptography.

15. It is the ratio of Deposits which banks have to keep with RBI. Under CRR a certain percentage of the
total bank deposits has to be kept in the current account with RBI. Banks don’t earn anything on that.
Banks will not have access to this amount. They cannot use this money for any of their economic or
commercial activities. Banks can’t lend this portion of money to corporate or individual borrowers. With
increase in CRR the lending capacity of banks will decrease as they will have less money.

16. The amount of money supply in the economy impacts prices i.e. when money supply increases
inflation increases and when money supply decreases inflation decreases.
Money supply impacts GDP also, as more money is required to increase the output. When the demand
for money increases, rate of interest goes up in the economy. So, when money supply increases then rate
of interest may cool/decrease in the economy and vice versa.
Created with by ForumIAS.com – The Knowledge Network for Civil Services.
Visit http//forumias.com New! http://forumiasacademy – Prelims & Mains Test Series

ForumIAS
PRELIMS MARATHON COMPILATION FOR THE MONTH OF JUNE, 2018

17. Key Provisions of FRDI bill: It seeks to establish a Resolution Corporation to replace the existing
Deposit Insurance and Credit Guarantee Corporation(DICGC) Resolution Corporation will monitor the
financial firms such as banks and insurance companies, anticipate their risk of failure, take corrective
action, and resolve them in case of such failure. The Corporation will also provide deposit insurance up
to a certain limit, in case of bank failure.

18. In national income, all new goods and services produced are included. So, expenditure on purchase of
an old house is not included. Brokerage on sale of shares is income of the agent for his services and it is
included. Meals given to the beggars in not included because it is expenditure without any productive
return. Commission on the sale on second hand car is again the income of the agent for his services and it
will be included.

19. Frictional unemployment is unemployment that occurs from the inevitable time delays in finding new
employment in a free market. It may also be called ‘search unemployment as it relates to the time taken to
search for new employment. For example, if you graduate from university, you can’t necessarily expect to
find a job straight away which matches your skills. This period of searching for a job is known as
frictional unemployment. Frictional unemployment will also occur when people are switching between
jobs, either because they have been made redundant or are looking for new employment.

20. If the real exchange rate is equal to one, currencies are at purchasing power parity.This means that
goods cost the same in two countries when measured in the same currency. If the real exchange rises
above one, this means that goods abroad have become more expensive than goods at home. The real
exchange rate is often taken as a measure of a country’s international competitiveness

21. National Pharmaceutical Pricing Authority [NPPA] is an independent body under Department of
Pharmaceuticals under Ministry of Chemicals and Fertilizers. Its functions are to fix/revise the controlled
bulk drugs prices and formulations. National List of Essential Medicines contains 376 medicines. Criteria
for inclusion into this list includes public health emergency, cost effective medicine etc. Once a drug or
medical device is included in NLEM, its price can be capped by NPPA. Recently, NPPA capped the
prices of knee implants. Surgical procedures are not covered under this.

22. Apart from currency notes and coins, the balance in savings, or current account deposits, heldby the
public in commercial banks is also considered money since cheques drawn on theseaccounts are used to
settle transactions. Such deposits are called demand deposits as they arepayable by the bank on demand
from the account holder.

23. Indian diaspora have been found to be world’s largest as per World Migration Report 2018.USA has
46.6 million international migrants

Created with by ForumIAS.com – The Knowledge Network for Civil Services.


Visit http//forumias.com New! http://forumiasacademy – Prelims & Mains Test Series

ForumIAS
PRELIMS MARATHON COMPILATION FOR THE MONTH OF JUNE, 2018

24. It is to provide comprehensive insurance coverage against crop loss. It is compulsory for farmers
availing crop loans for notified crops in notified areas and voluntary for non-loanee farmers.
Losses covered - Non-Preventable risk such as Natural Fire, Storm, Hailstorm, Cyclone and Inundation
has also been included as a localized calamity. Post Harvest losses also covered.

25. It is to promote organic farming and the products will be linked with the market. It will be
implemented in a cluster based approach and farmers will be funded to meet the expenditure from farm
to market. There is no liability for farmers for expenditure on certification.

26. NAM is a pan-India electronic trading portal which networks the existing APMC mandis to create a
unified national market for agricultural commodities. NAM is a virtual market but it is connected to a
physical market (mandi) at the back end and promotes genuine price discovery.

27. It is an ambitious programme to provide network infrastructure with affordable broadband


connectivity on a non-discriminatory basis to all households in the country. It aimed to realize the vision
of Digital India, in partnership with States and the Private Sector. The programme was previously called
as the National Optical Fibre Network, approved in October 2011.
The National Optical Fibre Network (NOFN) aims to connect all the 2,50,000 Gram panchayats in the
country and provide 100 Mbps connectivity to all gram panchayats (GPs).

Created with by ForumIAS.com – The Knowledge Network for Civil Services.


Visit http//forumias.com New! http://forumiasacademy – Prelims & Mains Test Series

ForumIAS
PRELIMS MARATHON COMPILATION FOR THE MONTH OF JUNE, 2018

Polity

Q.1) Which one of the following statements is income?


a) Goa attained full statehood in 1987
b) Diu is an island in the Gulf of Khambhat
c) Daman and Diu were separated from Goa by the 56th Amendment of the Constitution of India
d) Dadra and Nagar Haveli were under French colonial rule till 1954

Q.2) The parliament can make any law for the whole or any part of India for implementing International
treaties:
a) With the consent of all the States
b) With the consent of the majority of States
c) With the consent of the States concerned
d) Without the consent of any State

Q.3) Consider the following statements about the Attorney General of India:
1. He is appointed by the President of India.
2. He must have the same qualifications as are required for a judge of the Supreme Court.
3. He must be a member of either House of Parliament.
4. He can be removed by impeachment by Parliament.
Which of these statements are correct?
a) 1 and 2
b) 1 and 3
c) 2, 3 and 3
d) 3 and 4

Q.4) Consider the following functionaries:


1. Cabinet Secretary
2. Chief Election Commissioner
3. Union Cabinet Ministers
4. Chief Justice of India
Their Correct sequence, in the Order of Precedence is:
a) 3,4,2,1
b) 4,3,1,2
c) 4,3,2,1
d) 3,4,1,2

Q.5) the primary function of the Finance Commission in India is to


a) Distribute revenue between the Centre and the States
b) Prepare the Annual Budget
c) Advise the President on financial matters

Created with by ForumIAS.com – The Knowledge Network for Civil Services.


Visit http//forumias.com New! http://forumiasacademy – Prelims & Mains Test Series

ForumIAS
PRELIMS MARATHON COMPILATION FOR THE MONTH OF JUNE, 2018

d) Allocate funds to various ministers of the Union and State Governments.

Q.6) Which one of the following statements about a Money Bill is not correct?
a) A Money Bill can be tabled in either House of Parliament
b) The Speaker of LokSabha is the final authority to decide whether a Bill is a Money Bill or not
c) The RajyaSabha must return a Money Bill passed by the lokSabha and send it for consideration within
14 Days
d) The President cannot return a Money Bill to the lokSabha for reconsideration

Q.7) Consider the following statements regarding the National Human Rights Commission of India:
1. Its Chairman must be a retired Chief Justice of India.
2. It has formation in each state as State Human Rights Commission.
3. Its powers are only recommendatory in nature.
4. It is mandatory to appoint a woman as a member of the Commission.
Which of the above statements are correct?
a) 1, 2, 3 and 4
b) 2 and 4
c) 2 and 3
d) 1 and 3

Q.8) With reference to Ad hoc judges of the Supreme Court, consider the following statements:
1. They are appointed when there is a lack of quorum of permanent judges to hold a session of the
Supreme Court.
2. They are appointed by the President on recommendation of the Chief Justice of India.
3. A judge of high court cannot be appointed as an ad hoc judge.
Which of the statements given above is/are correct?
a) 1 and 2 only
b) 1 only
c) 2 and 3 only
d) 1, 2 and 3

Q.9) Consider the following statements about the Zonal Councils.


1. Zonal Councils have been established under the provision of Constitution.
2. For the purpose of creation of Zonal Councils, the territory of India has been divided into five zones.
3. The Zonal Councils provide an excellent forum where irritants between Centre and States and amongst
States can be resolved through free and frank discussions and consultations.
Which of the above statements are correct?
a) 1 & 2 Only
b) 2 & 3 Only
c) 1 & 3 Only
d) All of the above

Q.10) With reference to the Prorogation of a House of the Parliament, consider the following statements:
1. It terminates only a sitting and not a session of the house.
2. It is done by the presiding officer of the house.
Created with by ForumIAS.com – The Knowledge Network for Civil Services.
Visit http//forumias.com New! http://forumiasacademy – Prelims & Mains Test Series

ForumIAS
PRELIMS MARATHON COMPILATION FOR THE MONTH OF JUNE, 2018

3. It does not affect the bills pending before the house.


Which of the statements given above is/are correct?
a) 1 and 2 only
b) 2 only
c) 2 and 3 only
d) 3 only

Q.11) Jawaharlal Nehru was the chairman of which of the following committees of the constituent
assembly?
1. Union Powers Committee
2. States Committee
3. Union Constitution Committee
4. Rules of Procedure Committee
Select the correct answer using the code given below.
a) 1, 2 and 3 only
b) 3 and 4 only
c) 2, 3 and 4 only
d) 1, 2 and 4 only

Q.12) Disqualification on ground of defection does not apply in cases of


1. Political party mergers
2. Political party splits
3. A candidate quitting political party
Choose the correct answer from the codes below
a) 1 and 2 only
b) 2 and 3 only
c) 1 and 3 only
d) 2 only

Q.13)Which of the following category of Constitution amendments would require the ratification of the
states in India?
1. Citizenship clauses
2. Changes in the Concurrent List under Seventh Schedule.
3. Changes in the State boundaries
4. Changing the number of puisne judges in the Supreme Court
Choose the correct answer from the codes below?
a) 1 and 4 only
b) 2 and 3 only
c) 1, 2 and 3 only
d) 2 only

Q.14) Consider the following Statements about the speaker of a legislative assembly of a state in India:
1. He decides whether a bill is a Money bill and his decision on it is final
2. A resolution passed by a majority of two-third of all the then members of the house is required for his
removal from his office.
Created with by ForumIAS.com – The Knowledge Network for Civil Services.
Visit http//forumias.com New! http://forumiasacademy – Prelims & Mains Test Series

ForumIAS
PRELIMS MARATHON COMPILATION FOR THE MONTH OF JUNE, 2018

Select the correct answer using the codes below:


a) 1 only
b) 2 only
c) Both 1 and 2
d) Neither 1 nor 2

Q.15) under which of the following cases can the Governor use his individual discretion in appointing
Chief Minister?
1. When no political party has clear majority.
2. When the Chief Minister in office dies suddenly.
Select the correct answer using the code given below.
a) 1 only
b) 2 only
c) Both 1 and 2
d) Neither 1 nor 2

Q.16) Appointment of the Central Vigilance Commissioner is recommended by a committee consisting of


a) Prime Minister, Speaker of LokSabha, Deputy Chairperson of RajyaSabha, Home Minister, Leader of
Opposition in both the houses of parliament
b) Prime Minister, Home Minister, Leader of opposition in LokSabha
c) Prime Minister, Speaker of LokSabha, Home Minister, Leader of Opposition in LokSabha
d) Prime Minister as Chairperson, the Leader of Opposition in the LokSabha and a Union Cabinet
Minister nominated by the Prime Minister

Q.17) Consider the following about National Commission for Scheduled Caste
a) The commission presents an annual report to the Parliament.
b) It has all the powers of a criminal court trying a suit.
Which of the following are correct?
a) Only a
b) Only b
c) Both a and b
d) Neither a nor b

Q.18) Appointment of the Central Information Commissioner is recommended by a committee consisting


of
a) Prime Minister, Speaker of LokSabha, Deputy Chairperson of RajyaSabha, Home Minister, Leader of
Opposition in both the houses of parliament
b) Prime Minister, Home Minister, Leader of opposition in LokSabha
c) Prime Minister, Speaker of LokSabha, Home Minister, Leader of Opposition in LokSabha
d) Prime Minister as Chairperson, the Leader of Opposition in the LokSabha and a Union Cabinet
Minister nominated by the Prime Minister

Q.19) Joint State Public Service Commission created for two or more states is a
a) Statutory Body
b) Constitutional Body
Created with by ForumIAS.com – The Knowledge Network for Civil Services.
Visit http//forumias.com New! http://forumiasacademy – Prelims & Mains Test Series

ForumIAS
PRELIMS MARATHON COMPILATION FOR THE MONTH OF JUNE, 2018

c) Non-statutory body
d) None of the above
Q.20) Consider the following statements about the Central Information Commission
1. Central Information Commission is a constitutional body.
2. selection committee consisting of the Prime Minister, the Leader of Opposition in the LokSabha and a
Union Cabinet Minister nominated by the Prime Minister.
Which of the above statement(s) is/are correct?
(a) Only 1
(b) Only 2
(c) Both 1 & 2
(d) Neither 1 nor 2

Q.21) which of the following National Commissions / Central Bodies is not correctly matched
Commission / Body: Falls Under
a) Central Information Commission: Ministry of Personnel
b) Inter-State Council: Ministry of Home Affairs
c) National Commission for STs: Ministry of Social Justice & Empowerment
d) Finance Commission: Ministry of Finance

Q.22) Consider the following statements regarding the Central Bureau of Investigation of India:
1. The CBI is a statutory body.
2. Central Bureau of Investigation (CBI) is under the Ministry of Home Affairs.
3. The establishment of the CBI was recommended by the Santhanam Committee on Prevention of
Corruption
4. It derives its powers from the Delhi Special Police Establishment Act, 1946
Select the correct answer using the code given below:
a) 3 and 4 only
b) 1, 2 and 4 only
c) 2, 3 and 4 only
d) 1, 2, 3 and 4
Q.23) The institution of Ombudsman was first created in
a) Sweden
b) Newzeland
c) Finland
d) England

Q.24) which of the following legal and institutional framework to check corruption and redress citizens’
grievances in India
1. Public Servants (Enquiries) Act,
2. Indian Penal Code,
3. Parliament
4. Delhi Police Establishment Act,
5. National Commission for SCs
6. National Commission for STs,
Select the correct answer using the code given below:
Created with by ForumIAS.com – The Knowledge Network for Civil Services.
Visit http//forumias.com New! http://forumiasacademy – Prelims & Mains Test Series

ForumIAS
PRELIMS MARATHON COMPILATION FOR THE MONTH OF JUNE, 2018

a) 1, 2, 3 and 4 only
b) 1,2,4,5 and 6 only
c) 1, 2 and 4 only
d) 1, 2, 3, 4, 5 and 6

Q.25) Consider the following statements regarding the The Official Language Act:
1. The Official Language Act (1963) lays down that English should be used for purposes of
communication between the Union and the non- Hindi states
2. Hindi is used for communication between a Hindi and a non-Hindi state, such communication in
Hindi should be accompanied by an English translation.
Which of the above statement(s) is/are correct?
a) Only 1
b) Only 2
c) Both 1 & 2
d) Neither 1 nor 2
Q.26) which of the following languages are not granted the classical language status.
1. Tamil
2. Assamese
3. Telugu
4. Kannada

Q.27) Consider the following about all India services


1. The All-India Services Act authorised the Central government to make rules in consultation with the
state governments for the regulation of recruitment and service conditions of the members of all-India
services...
2. Members of these services are recruited and trained by the Central government and assigned to center
only for work.
Which of the above statement(s) is/are correct?
a) Only 1
b) Only 2
c) Both 1 & 2
d) Neither 1 nor 2

Q.28) Consider the following statements regarding the Central Vigilance Commission of India:
1. The Chairman CVC are appointed by President on the recommendation of a three member committee
consisting of the Prime Minister as its head, Central Home Minister and leader of opposition in
LokSabha.
2. The Chairman hold office for a term of five years or until they attain the age 65 years whichever is
earlier.
3. It has all powers of civil court.
Select the correct answer using the codes below:
a) 1 and2 only
b) 2 and 3 only
c) 1 and 3 only
d) 1, 2 and 3
Created with by ForumIAS.com – The Knowledge Network for Civil Services.
Visit http//forumias.com New! http://forumiasacademy – Prelims & Mains Test Series

ForumIAS
PRELIMS MARATHON COMPILATION FOR THE MONTH OF JUNE, 2018

Answer Key

Qs. Ans. Qs. Ans. Qs. Ans.


1 D 11 A 21 C
2 D 12 A 22 A
3 A 13 D 23 A
4 C 14 A 24 D
5 A 15 C 25 C
6 A 16 B 26 B
7 D 17 D 27 A
8 B 18 D 28 C
9 B 19 A
10 D 20 B

Explanation

1. Dadra and Nagar Haveli were under Portuguese rule till 1961

2. According to art 253 which is concerned with legislation for giving effect to international agreement,
parliament has the power to make any law for the whole or any part of the territory of India for
implementing any treaty, agreement or convention with any other country or countries or any decision
made at international conference, association or other body.

3. Under art.76, the Attorney General is appointed by the President. He has the qualification of a judge of
Supreme Court. He holds office during the pleasure of the president. He is not a Member of Parliament,
as his office is an office of profit.

4. In the protocol: CJI -6th, Cabinet Ministers-8th, CEC-9th, Cabinet Secretary-11th position.

5. Under Art.280, the Finance Commission is to advise on revenue sharing between Union and State other
functions listed here are done by Finance Ministry.

6. A Money Bill can't be introduced in RajyaSabha. A Money Bill can be table in the LokSabha only. A bill
shall be deemed to be a Money Bill if it contains only provisions dealing with all or any of the matters
mentioned under Article 110 of the Constitution.

7. In keeping with the spirit of human rights movement all over the world, the national Human Rights
Commission (NHRC) came into existence in India in 1993 through an ordinance promulgated on 28th
September 1993 by the president of India. The ordinance was replaced by a statute called the Protection of
Human Rights Act, 1993 which came into force in 1994. This Act provides for setting up the NHRC at the
center as well as one commission each at the state level. Chairman of NHRC must be no less than a
Created with by ForumIAS.com – The Knowledge Network for Civil Services.
Visit http//forumias.com New! http://forumiasacademy – Prelims & Mains Test Series

ForumIAS
PRELIMS MARATHON COMPILATION FOR THE MONTH OF JUNE, 2018

former Chief Court. Its powers are only recommendatory in nature. Only 15 states have state human
Rights Commission and it is not mandatory to appoint a women as a member of the commission.

8. When there is a lack of quorum of the permanent judges to hold or continue any session of the
Supreme Court, the Chief Justice of India can appoint a judge of a High Court as an ad hoc judge of the
Supreme Court for a temporary period. He can do so only after consultation with the chief justice of the
High Court concerned and with the previous consent of the president. The judge so appointed should be
qualified for appointment as a judge of the Supreme Court. It is the duty of the judge so appointed to
attend the sittings of the Supreme Court, in priority to other duties of his office. While so attending, he
enjoys all the jurisdiction, powers and privileges (and discharges the duties) of a judge of the Supreme
Court

9. Zonal Councils have been established by the State Reorganization Act, 1956.

10. Prorogation terminates both the sitting and the session of the house. It is done by the President of
India. It does not affect the bills pending before the house. However all pending notices (other than those
for introducing bills) lapse on prorogation and fresh notices have to be given for the next.

11. Jawaharlal Nehru was the Chairman of:


1. Union Powers Committee
2. States Committee
3. Union Constitution Committee
The Rules of Procedure Committee was chaired by Dr. Rajendra Prasad.

12. The grounds for disqualification under theAnti-Defection Laws are as follows.
(a) If an elected member voluntarily gives up his membership of a political party;
(b)If he votes or abstains from voting in such House contrary to any direction issued byhis political party
or anyone authorised to do so, without obtaining prior permission.
As a pre-condition for his disqualification, his abstention from voting should not be condoned by his
party or the authorised person within 15 days of such incident. Finally the 91st Constitutional
Amendment Act, 2003, changed this. So now at least two-thirds of the members of a party have to be in
favour of a "merger" for it to have validity in the eyes of the law. "The merger of the original political
party or a member of a House shall be deemed to have taken place if, and only if, not less than two thirds
of the members of the legislature party concerned have agreed to such merger," states the Tenth Schedule.

13. Citizenship clauses need a simple majority only in the Parliament. The same is true for changing the
boundary of states and for changing number of judges.

14. When the Speaker is removed from office by a resolution which is passed by a majority of all the
members of the House. While such a process is underway, the Speaker cannot preside over the House,
but can take part in the proceedings of the House.

15. When no party has a clear majority in the legislative assembly, then governor may exercise his
personal discretion in the selection and appointment of CM. In such a situation the governor usually

Created with by ForumIAS.com – The Knowledge Network for Civil Services.


Visit http//forumias.com New! http://forumiasacademy – Prelims & Mains Test Series

ForumIAS
PRELIMS MARATHON COMPILATION FOR THE MONTH OF JUNE, 2018

appoints, the leader of largest party or largest coalition party, as the CM and ask him to seek a vote of
confidence in the house within a month.
The governor may exercise his individual judgment in the selection and appointment of CM when CM in
the office dies suddenly and there is no obvious successor. However, on the death of a CM, the ruling
party usually elects a new leader and governor appoints him/her as CM.

16. The CVC is a multi-member body consisting of a Central Vigilance Commissioner (chairperson) and
not more than two vigilance commissioners. They are appointed by the president by warrant under his
hand and seal on the recommendation of a three-member committee consisting of the prime minister as
its head, the Union minister of home affairs and the Leader of the Opposition in the LokSabha.

17. Statement a) Report of the Commission


The Commission presents an annual report to the President. It can also submit a report as and when it
thinks necessary. The President places all such reports before the Parliament, along with a memorandum
explaining the action taken on the recommendations made by the Commission. The memorandum
should also contain the reasons for the non-acceptance of any of such recommendations.
Statement b) Powers of the Commission
The Commission is vested with the power to regulate its own procedure. The Commission, while
investigating any matter or inquiring into anycomplaint, has all the powers of a civil court trying a suit
and in particular in respect of the following matters:
(a) Summoning and enforcing the attendance of any person from any part of India and examining him on
oath;
(b)Requiring the discovery and production of any document;
(c) Receiving evidence on affidavits;
(d) Requisitioning any public record from any court or office
(e) Issuing summons for the examination of witnesses and documents; and
(f) Any other matter which the President may determine.

18. The Commission consists of a Chief Information Commissioner and not more than ten Information
Commissioners. They are appointed by the President on the recommendation of a committee consisting
of the Prime Minister as Chairperson, the Leader of Opposition in the LokSabha and a Union Cabinet
Minister nominated by the Prime Minister. They should be persons of eminence in public life with wide
knowledge and experience in law, science and technology, social service, management, journalism, mass
media or administration and governance. They should not be a Member of Parliament or Member of the
Legislature of any State or Union Territory. They should not hold any other office of profit or connected
with any political party or carrying on any business or pursuing any profession.

19. The Constitution makes a provision for the establishment of a Joint State Public Service Commission
(JSPSC) for two or more states. While the UPSC and the SPSC are created directly by the Constitution, a
JSPSC can be created by an act of Parliament on the request of the state legislatures concerned. JSPSC is a
statutory and not a constitutional body.

20. The Central Information Commission was established by the Central Government in 2005. It was
constituted through an Official Gazette Notification under the provisions of the Right to Information Act
(2005).Hence, it is not a constitutional body.
Created with by ForumIAS.com – The Knowledge Network for Civil Services.
Visit http//forumias.com New! http://forumiasacademy – Prelims & Mains Test Series

ForumIAS
PRELIMS MARATHON COMPILATION FOR THE MONTH OF JUNE, 2018

They are appointed by the President on the recommendation of a committee consisting of the Prime
Minister as Chairperson, the Leader of Opposition in the LokSabha and a Union Cabinet Minister
nominated by the Prime Minister.

21. National Commission for SCs: Ministry of Social Justice & Empowerment
National Commission for STs: Ministry of Tribal Affairs

22. Central Bureau of Investigation (CBI) was set up in 1963 by a resolution of the Ministry of Home
Affairs. Later, it was transferred to the Ministry of Personnel and now it enjoys the status of an attached
office. The Special Police Establishment (which looked into vigilance cases) setup in 1941 was also
merged with the CBI. The establishment of the CBI was recommended by the Santhanam Committee on
Prevention of Corruption (1962-1964). The CBI is not a statutory body. It derives its powers from the
Delhi Special Police Establishment Act, 1946.

23. The institution of Ombudsman was first created in Sweden in 1809.‘Ombud’ is a Swedish term and
refers to a person who acts as the representative or spokesman of another person. From Sweden, the
institution of Ombudsman spread to other Scandinavian countries—Finland (1919), Denmark (1955) and
Norway (1962).

24. The existing legal and institutional framework to check corruption and
Redress citizens’ grievances in India consists of the followings:
1. Public Servants (Enquiries) Act, 1850
2. Indian Penal Code, 1860
3. Special Police Establishment, 1941
4. Delhi Police Establishment Act, 1946
5. Prevention of Corruption Act, 1988
6. Commissions of Inquiry Act, 1952 (against political leaders and eminent
Public men)
7. All-India Services (Conduct) Rules, 1968
8. Central Civil Services (Conduct) Rules, 1964
9. Railway Services (Conduct) Rules, 1966
10. Vigilance organisations in ministries / departments, attached and
Subordinate offices and public undertakings
11. Central Bureau of Investigation, 1963
12. Central Vigilance Commission, 1964
13. State Vigilance Commissions, 1964
14. Anti-corruption bureaus in states
15. Lokpal (Ombudsman) at the Centre
16. Lokayukta (Ombudsman) in states
17. Divisional Vigilance Board
18. District Vigilance Officer
19. National Consumer Disputes Redressal Commission
20. National Commission for SCs
21. National Commission for STs
22. Supreme Court and High Courts in states
Created with by ForumIAS.com – The Knowledge Network for Civil Services.
Visit http//forumias.com New! http://forumiasacademy – Prelims & Mains Test Series

ForumIAS
PRELIMS MARATHON COMPILATION FOR THE MONTH OF JUNE, 2018

23. Administrative Tribunals (quasi-judicial bodies)


24. Directorate of Public Grievances in the Cabinet Secretariat, 1988
25. Parliament and its committee

25. The Official Language Act (1963) lays down that English should be used for purposes of
communication between the Union and the non-Hindi states (that is, the states that have not adopted
Hindi as their official language). Further, where Hindi is used for communication between a Hindi and a
non-Hindi state, such communication in Hindi should be accompanied by an English translation.

26. 1. Tamil 2004


2. Sanskrit 2005
3. Telugu 2008
4. Kannada 2008
5. Malayalam 2013
6. Odia 2014

27. The All-India Services Act of 1951 authorised the Central government to make rules in consultation
with the state governments for the regulation of recruitment and service conditions of the members of all-
India services. The members of these services are recruited and trained by the Central government but
are assigned to different states for work. They belong to different state cadres; the Centre having no cadre
of its own in this regard.

28. The Chairman and members hold office for a term of four years or until they attain the age 65 years
whichever is earlier.

Created with by ForumIAS.com – The Knowledge Network for Civil Services.


Visit http//forumias.com New! http://forumiasacademy – Prelims & Mains Test Series

ForumIAS
PRELIMS MARATHON COMPILATION FOR THE MONTH OF JUNE, 2018

Science and Technology


Q.1) The only snake that builds a nest is
a) Chain viper
b) King Cobra
c) Krait
d) Saw scaled viper

Q.2) The normal temperature of human body on the Kelvin scale is


a) 280
b) 290
c) 300
d) 310

Q.3) Most of the desert plants bloom during night time because
a) their blooming is controlled by low temperature
b) they are sensitive to the phases of moon
c) the desert insect eat away flowers during day time
d) the desert insects are active during night time

Q.4) Which of the following elements are present in all proteins?


1. Carbon
2. Hydrogen
3. Oxygen
4. Nitrogen
a) 2 and 3
b) 1,2 and 4
c) 1,3 and 4
d) 1,2,3 and 4

Q.5) Which one of the following hormones contains iodine?


a) Thyroxine
b) Testosterone
c) Insulin
d) Adrenaline

Q.6) Optical fibre works on the principle of


a) total internal reflection
b) refraction
c) scattering
d) interference

Created with by ForumIAS.com – The Knowledge Network for Civil Services.


Visit http//forumias.com New! http://forumiasacademy – Prelims & Mains Test Series

ForumIAS
PRELIMS MARATHON COMPILATION FOR THE MONTH OF JUNE, 2018

Q.7) An air bubble in water will act like


a) convex Mirror
b) convex lens
c) concave mirror
d) concave lens

Q.8) Match the following Vitamins with their deficiencies:


a) Vitamin A 1) Beri-beri
b) Vitamin B 2) Rickets
c) Vitamin C 3) Scurvy
d) Vitamin D 4) Night-Blindness
Select for the codes given below:
a) a-1, b-4, c-3, d-2
b) a-4, b-1, c-2, d-3
c) a-4, b-1, c-3, d-2
d) a-1, b-4, c-2, d-3

Q.9) Match the following Vitamins with their scientific names:


a) Vitamin B1 1) Cyanocobalamin
b) Vitamin B2 2) Riboflavin
c) Vitamin B3 3) Thiamine
d) Vitamin B12 4) Niacin
Select for the codes given below:
a) a-2, b-3, c-4, d-1
b) a-3, b-2, c-4, d-1
c) a-2, b-3, c-1, d-4
d) a-3, b-2, c-1, d-4

Q.10) Consider the following statements:


1. It is an indigenously developed nuclear capable missile.
2. It is a short range surface to surface missile.
3. It is the first missile to be developed by DRDO under Integrated Guided Missile Development
Programme.
The above properties refers to which of the following missiles?
a) Prithvi II
b) Brahmos
c) Agni
d) Akash

Q.11) Which of the following projects is/are related to providing last mile internet connectivity?
1. Project Loon
2. Aquila
3. White Fi
Select the correct answer using the code given below.
a) 1 only
Created with by ForumIAS.com – The Knowledge Network for Civil Services.
Visit http//forumias.com New! http://forumiasacademy – Prelims & Mains Test Series

ForumIAS
PRELIMS MARATHON COMPILATION FOR THE MONTH OF JUNE, 2018

b) 1 and 3 only
c) 2 and 3 only
d) 1, 2 and 3

Q.12) With reference to ‘Aquila’, recently in the news, which of the following statement is/are correct?
1. It is an initiative of Google launched in collaboration with Tata Trusts to educate women in rural India
on the benefits of using the Internet.
2. Rather than use under-ground fibre optic cables or construct cell towers to connect users, it will use
balloons that float in the stratosphere.
Select the correct answer using the code given below.
a) 1 only
b) 2 only
c) Both 1 and 2
d) Neither 1 nor 2

Q.13) Which of the following statement(s) is/are correct regarding ‘ketosis’ a metabolic process during
fasting?
1. During ketosis body burns stored fat for energy.
2. It is commonly observed in patients with diabetes.
3. It results build-up of acids called ketones within the body.
Which of the above given statements is/are correct?
a) 1 only
b) 1 and 3 only
c) 2 only
d) All the above

Q.14) The fundamental objective of proposed ELISA project of European Space Agency is to detect:
a) Gravitational Waves
b) Dark Matter and Dark Energy
c) Higgs Boson
d) Sterile Neutrinos

Q.15) Match List-I with List-II and select the correct answers from the codes given below:
List-I List-II
(Organization) (Location)
1. ISRO Thiruvanthapuram
2. IUCAA Pune
3. IUAC Bengaluru
4. VSSC New Delhi
a) A3 B2 C4 D1
b) A1 B2 C3 D4
c) A2 B4 C1 D3
d) A3 B1 C2 D4
Created with by ForumIAS.com – The Knowledge Network for Civil Services.
Visit http//forumias.com New! http://forumiasacademy – Prelims & Mains Test Series

ForumIAS
PRELIMS MARATHON COMPILATION FOR THE MONTH OF JUNE, 2018

Q.16) Scientists of Britain have built a “Gravity Tractor”. Identify the same from the following:
a) A double storeyed “Jugaad” type tractor trolley to be used in public transport.
b) A tractor with a mechanism to sweep public roads and is economical to run
c) A bullet train run on the principle of earth’s gravity with the help of powerful magnets
d) A space-craft with a mechanism which would check asteroids from hitting the earth.

Q.17) Which one of the following is not correctly matched?


a) Akash — A medium range multi-target missile
b) Nag — An anti-tank missile
c) Pinaka — A multi-barrel rocket launcher weapon system
d) Trishul—A short range surface to surface missile

Q.18) The earth’s satellite is kept moving in its orbit. It is due to the phenomenon of centripetal force
provided by
a) the rocket engine propelling the satellite
b) the gravitational attraction of the earth on the satellite
c) the gravitational attraction of the sun on the satellite
d) the gravitational attraction of satellite on earth

Q.19) Multiple Independent Re-entry Vehicles (MIRVs) are used in Agni-V missile made by DRDO,
consider the following statements in regard to
1. Under it, each missile will be capable of carrying 2-10 separate nuclear
2. Under it, each warhead can be assigned to a different
3. Under it, two or more warheads can be assigned to the same
Which of the following statements is/are correct?
a) 1 only
b) 2 only
c) 2 and 3 only
d) 1, 2 and 3

Q.20) Consider the following statements regarding Indian polar research station:
India established a research station named “Bharti” at Ny-Alesund in Svalbard region of
India established a research station named “Himadri” in the Larsemann Hills region of East
Which of the statements given above is/are correct?
a) 1 only
b) 2 only
c) Both 1 and 2
d) Neither 1 nor 2

Q.21 What is a keyboard used in a computer for?


a) To input text and numbers and send commands to the computer.
b) To create new keys to use with your computer.
c) To open the computer up.
d) To create pictures and images and send them to your computer.
Created with by ForumIAS.com – The Knowledge Network for Civil Services.
Visit http//forumias.com New! http://forumiasacademy – Prelims & Mains Test Series

ForumIAS
PRELIMS MARATHON COMPILATION FOR THE MONTH OF JUNE, 2018

Q.22) What is the scientific reason behind the usage of CO2 to extinguish the fire caused due to electrical
or other fire accidents?
a. It reduces the heat generation around the accident spot
b. CO2 reacts with oxygen thereby reduces the source for the flame.
c. It is heavier than oxygen and covers fire like a blanket
d. The lighter nature of CO2 makes it to act as a catalyst to extinguish the fire

Q.23) Which of the following statements is/are correct?


1. Mechanical wave doesn‘t require a medium for propagation.
2. Electromagnetic waves require medium for propagation.
Select the correct answer using the code given below:
a. 1 only
b. 2 only
c. Both 1 and 2
d. Neither 1 nor 2

Q.24) Recently people around the world have observed a rare celestial event, ―Supermoon. With respect
to this rare phenomenon consider the following statements:
1. Supermoon phenomenon is reported when the moon in the circular orbit comes close to the
orbit of the earth.
2. Supermoon occurs either during the full moon day or during the new moon day.
3. Technically this phenomenon is called as Apogee-Syzygy position.
4. Supermoon is free from the occurrence of eclipse.
Which of the above given statements above is/are correct?
a . 2 only
b . 1 and 3 only
c . 1, 2 and 4 only
d . 2, 3 and 4 only

Q.25) The term ―Dark matter‖ refers to unidentified type of matter comprising approximately about one
fourth of the mass and energy in the observable universe. With respect to this ―Dark matter‖ which one
among the following statement is not correct?
a . Dark matter doesn‘t obey the principle of electromagnetism.
b . Dark matters possess strong interaction with the normal matter.
c . Dark matter holds slow moving and weak self-interacting particles.
d . Dark matters tend to form non luminous stable halos around the galaxies.

Q.26) Consider the following statements with respect to the new Remote Sensing Data Policy:
1. Agencies other than ISRO can also be nodal agencies for remote sensing.
2. All remote sensing imagery and data up to one meter resolution will be made freely available.
3. The old 2001 policy mandated that ISRO could release only data up to 6.8 resolutions.
Which of the statements given above are correct?
a) 1 and 2 only
b) 1 and 3 only
Created with by ForumIAS.com – The Knowledge Network for Civil Services.
Visit http//forumias.com New! http://forumiasacademy – Prelims & Mains Test Series

ForumIAS
PRELIMS MARATHON COMPILATION FOR THE MONTH OF JUNE, 2018

c) 2 and 3 only
d) 1, 2 and 3

Q.27) The new version of advanced television, beyond the high definition television is 4 K television.
Consider the following statements in this regard?
1. The basic principle of 4K TV is Near Field
Communication (NFC).
2. The term 4K refers to the horizontal resolution of the images, which are all on the order of 4,000 pixels.
3. If the screen is too big then the HD gives the blurred images because the resolution of the images and
pixels are limited.
Which of the statements given above are correct?
a) 1 and 2 only
b) 2 and 3 only
c) 1 and 3 only
d) 1, 2 and 3

Q.28) What is the difference between present Wi – Fi technology and super Wi – Fi?
1. Present Wi – Fi technology supports 802.11 b/g/n standard while new super Wi – Fi will support
802.11 ad standard.
2. Super Wi – Fi can send data upto the length of 1 – 2 GB and eventually there would be no log in the
speed.
3. The new super Wi – Fi would function at the speed of 7 gigabytes per second, much more than the
speed of ordinary Wi – Fi.
Select the correct answer using the codes given below:
a) 1 and 2 only
b) 2 and 3 only
c) 1 and 3 only
d) 1, 2 and 3

Answer Key

Qs. Ans. Qs. Ans. Qs. Ans.


1 B 11 D 21 A
2 D 12 D 22 C
3 D 13 D 23 D
4 D 14 A 24 A
5 A 15 A 25 B
6 A 16 D 26 A
7 D 17 D 27 B
8 C 18 B 28 D
9 B 19 D
10 A 20 D
Created with by ForumIAS.com – The Knowledge Network for Civil Services.
Visit http//forumias.com New! http://forumiasacademy – Prelims & Mains Test Series

ForumIAS
PRELIMS MARATHON COMPILATION FOR THE MONTH OF JUNE, 2018

Explanaiton

1. King Cobra is the only snake that builds a nest made of leaves and branches and guards it until the
eggs hatch.

2. Normal body temperature of human is 37 But. When we convert 37 into Kelvin it becomes 310 K
because 0 = 0 +273 = 273 K
37 =37°+ 273=310 K

3. Most of the desert plants bloom during night time because of very high temperature in day time, the
desert insects are active during night time which help in pollination

4. All proteins are formed of amino acids that contains elements viz. Carbon, Hydrogen, oxygen and
Nitrogen.

5. Thyroxine contains iodinated amino acid (tyrosine) Thyroxine hormone released by Thyroid glands.
The deficiency of iodine causes ‘Goitre’ disease by the enlargement of Thyroid glands .

6. Optical fibres works on the principle of total internal reflection (TIR), when light in incident at an angle
greater than critical angle at one end, it gets refracted and gets incident on the interface of fibere and its
coating. This makes light to suffer multiple total internal reflections.

7. An air bubble in water will act like a concave e lens because the index of refraction of air is lens than
that of water

8. Deficiency of following vitamins causes the given diseases:


Vitamin A – Night-Blindness
Vitamin B – Beri-beri
Vitamin C – Scurvy
Vitamin D - Rickets

9. Scientific Names of following Vitamins are:


Vitamin A - Retinol
Vitamin B1 – Thiamine
Vitamin B2 – Riboflavin
Vitamin B3 – Niacin
Vitamin B6 – Pyridoxine
Vitamin B12 - Cyanocobalamin
Vitamin C – Ascorbic Acid
Vitamin D – Cholecalciferol
Vitamin E - Tocopherol
Vitamin K - Phylloquinone

Created with by ForumIAS.com – The Knowledge Network for Civil Services.


Visit http//forumias.com New! http://forumiasacademy – Prelims & Mains Test Series

ForumIAS
PRELIMS MARATHON COMPILATION FOR THE MONTH OF JUNE, 2018

10. Prithvi II is an indigenously developed nuclear capable missile. With a strike range of 350 km, the
Prithvi-II is capable of carrying 500 kg to 1,000 kg of warheads and is thrusted by liquid propulsion twin
engines. It uses advanced inertial guidance system with manoeuvring trajectory to hit its target. It is a
short range surface to surface missile.
Inducted into Indian armed forces in 2003, the nine-metre-tall, single-stage liquid-fuelled Prithvi-II is the
first missile to be developed by the DRDO under the Integrated Guided Missile Development
Programme.

11. Project Loon is a research and development project being developed by X (formerly Google X) with
the mission of providing Internet access to rural and remote areas. Aquila by Facebook is a solar power
plane that beams internet connectivity from the Sky. White Fi by Microsoft uses unused spectrum to
provide connectivity.

12. Aquila is the code name of the unmanned solar-powered drone developed by Facebook. The company
plans to use a linked network of the drones to provide internet access to large rural areas. Recently
Facebook completed the first successful flight of Aquila. However, as with its Internet.org project,
Facebook will not be dealing with customers directly, instead partnering with local ISPs to offer the
services.

13. All the statements are correct. Ketosis is a normal metabolic process. When the body does not have
enough glucose for energy, it burns stored fats instead; this results in a build-up of acids called ketones
within the body.
Some people encourage ketosis by following a diet called the ketogenic or low-carb diet. The aim of the
diet is to try and burn unwanted fat by forcing the body to rely on fat for energy, rather than
carbohydrates.
Ketosis is also commonly observed in patients with diabetes, as the process can occur if the body does not
have enough insulin or is not using insulin correctly.

14. ELISA Pathfinder is paving the way for future missions by testing in flight the very concept of
gravitational wave detection: it will put two test masses in a near-perfect gravitational free-fall and
control and measure their motion with unprecedented accuracy. It's objective is to detect thegravitational
waves and not the dark matter and Dark Energy. LISA Pathfinder is an ESA mission, which also carries a
NASA payload. Only feasible in space LISA pathfinder is a pioneer mission. Not only are these
technologies new, they cannot be properly verified on the ground. This is because Earth's gravity and
environment would hamper the test results.

15. Indian Space Research Organisation (ISRO) is headquartered in Bengaluru. The Inter–University
Centre for Astronomy and Astrophysics (IUCAA) is located in Pune, India.
Inter–University Accelaerator Centre (IUAC) is an autonomous research facility of University Grants
Commission which is based in New Delhi. Vikram Sarabhai Space Centre (VSSC) is a major space
research centre of the Indian Space Research Organisation, focusing on rocket and space vehicles, is
located in Thiruvananthapuram, Kerala.

Created with by ForumIAS.com – The Knowledge Network for Civil Services.


Visit http//forumias.com New! http://forumiasacademy – Prelims & Mains Test Series

ForumIAS
PRELIMS MARATHON COMPILATION FOR THE MONTH OF JUNE, 2018

16. ‘Gravity tractor’ (GT) is a spacecraft with a mechanism that can deflect another object in space without
physically contacting it. It will use only its gravitational field to transmit the required impulse. It can
effect a potentially hazardous asteroid that might hit earth.

17. Akash is a medium–range mobile surface–to– air missile. Nag is an antitank missile by DRDO. Pinaka
is a multiple rocket launcher produced for Indian Army by DRDO while Trishul is a missile system of a
short range surface to–air missile as a part of the Integrated Guided Missile Development Program.

18. An orbiting satellite is a projectile, that the only force acting upon an oribiting satellite is the
force of gravity. The force of gravity also accelerates it towards the earth. But the earth’s round shape
prevents it to fall on ground and if the satellite is in more than 8000 m/s speed it also contributes to the
maintenance of motion in a particular orbit or elliptical path.

19. Multiple Independent Re–entry Vehicle (MIRV) is a ballistic missile payload or “bus” of a ballistic
missile. It is capable of being aimed to hit one of a group of targets. A unitary warhead is a single
warhead on a single missile Multiple re-entry vehicle (MRV) is an intermediate case in which the
MRV missile carries several reentry vehicles, which can be dispersed but not individually aimed.

20. Both the statements regarding Indian Polar Research stations are incorrect.
India’s first expedition to Antarctica was in 1981, she expanded it by undertaking its first Arctic
expedition in August 2007 by setting up ‘Himadri’ at Nye–Alesund in Norway in 2008 to mark the
International Polar Year (IPY). The first Antarctic research station was Gangotri (1983) and second was
Maitri (1989). ‘Bharti’ or ‘Bharthi’ is the third Antarctic station at Larsemann in East Antarctica.

21. A computer keyboard, a typewriter–style device having a particular arrangement of buttons or keys,
is a main input device. It inputs text, numbers and sends commands to the computer.

22. Carbon dioxide extinguishers work because carbon dioxide molecules are heavier than oxygen
molecules. When the liquid carbon dioxide inside the extinguisher expands into a gas, the carbon dioxide
pushes out any oxygen surrounding the fire. Without oxygen available to fuel the chemical reaction, the
fire goes out quickly. This property can also make them dangerous in confined spaces since the carbon
dioxide also pushes out any oxygen that the user needs to breathe.

23. Sound waves in other terms to be called as mechanical a wave which compulsorily requires a medium
for its propagation. Whereas, Electromagnetic waves doesn‘t depends with the medium, which can even
travel in the vacuum.

24. Asupermoon is the coincidence of a full moon or a new moon with the closest Asupermoon is the
coincidence of a full moon or a new moon with the closest approach that the Moon makes with the Earth
on its elliptical orbit. If a supermoon falls on a full moon day then it results in the largest apparent size of
the lunar disk for an observer on the Earth. The technical name is the perigee-syzygy of the Earth–Moon–
Sun system. Occasionally, the supermoon coincides with the total lunar eclipse.

Created with by ForumIAS.com – The Knowledge Network for Civil Services.


Visit http//forumias.com New! http://forumiasacademy – Prelims & Mains Test Series

ForumIAS
PRELIMS MARATHON COMPILATION FOR THE MONTH OF JUNE, 2018

25. Dark matter is a hypothetical material that an astronomer or scientist cannot observe through
ordinary telescopes. Although dark matter has not been directly observed, its existence and properties are
inferred from its gravitational effects such as the motions of visible matter. The name refers to the fact
that it does not emit or interact with electromagnetic radiation, such as light, and is thus invisible to the
entire electromagnetic spectrum. It is considered to be responsible for holding all the normal matter in the
universe together. These dark matter particles have to be quite massive to form the observed structures,
about as heavy as the heaviest particles we know already. If dark matter particles weren‘t heavy enough
they wouldn‘t clump sufficiently, which is why they are called WIMPs for ―Weakly Interacting Massive
Particles. And we know that neutrinos, even though weakly interacting, can‘t make up dark matter
because they are too light and they wouldn‘t clump strongly enough to seed galaxy filaments.

26. The old 2001 policy mandated that ISRO could release only data upto 5.8 meter resolution.

27. 4K TV is an Ultra high definition television. UHDTV, has horizontal resolution on the order of 4,000
pixels. Since its screen is too big the HD gives blurred images.

28. New Super Wi-Fi will support 802.11 ad standard, it can send data upto the length
of 1-2 GB and would function at the speed of 7 gigabits per second.

Created with by ForumIAS.com – The Knowledge Network for Civil Services.


Visit http//forumias.com New! http://forumiasacademy – Prelims & Mains Test Series

ForumIAS
PRELIMS MARATHON COMPILATION FOR THE MONTH OF JUNE, 2018

Geography and Environment

Q.1) The concept of carbon credit originated from which one of the following?
a) Earth Summit, Rio de Janeiro
b) Kyoto Protocol
c) Montreal protocol
d) G-8 Summit Heiligendamm

Q.2) The United Nations Framework Convention on climatic change (UNFCCC) is an international treaty
drawn at
a) United Nations Conference on the Human environment, Stockholm, 1972
b) UN conference on environment and development, Rio de Janeiro, 1992
c) World summit on sustainable Development Johannesburg, 2002
d) UN Climate change conference, Copenhagen, 2009

Q.3) consider the following which can be found in the ambient atmosphere:
1. Soot
2. Sulphur hexafluoride
3. Water Vapour
Which of the above contribute to the warming up of the atmosphere?
a) 1 and 2 only
b) 3 only
c) 2 and 3 only
d) 1,2 and 3

Q.4) Sustainable development is described as the development that meets the needs of the present
without compromising the ability of future generations to meets their own needs. In this perspective,
inherently the concept of sustainable development is intertwined with which of the following concepts?
a) Social justice & empowerment
b) Inclusive growth
c) Globalization
d) Carrying capacity

Q.5. Consider the following crops:


1. Cotton
2. Groundnut
3. Maize
4. Mustard

Created with by ForumIAS.com – The Knowledge Network for Civil Services.


Visit http//forumias.com New! http://forumiasacademy – Prelims & Mains Test Series

ForumIAS
PRELIMS MARATHON COMPILATION FOR THE MONTH OF JUNE, 2018

Which of the above are Khalif crops?


a) 1 and 2
b) 1,2 and 3
c) 3 and 4
d) 1,2,3 and 4

Q.6 Which one of the following statements is not correct?


a) The largest Buddhist monastery in India is in Assam
b) The language Konyak is spoken in Nagland
c) The largest river island in the world is in Assam
d) Sikkim is the least populated state of the Indian Union

Q.7. Consider the following statements:


1. Damodar Valley Corporation is the first multipurpose river valley project of independent India.
2. Damodar Valley Corporation includes thermal and gas power stations.
Which of the statements given above is/are correct?
a) 1 only
b) 2 only
c) Both 1 and 2
d) Neither 1 nor 2

Q.8) Consider the following statements:


1. The boundaries of a national park are defined by legislation.
2. A biosphere Reserve is declared a few specific species of flora and fauna.
3. In a wildlife Sanctuary, limited biotic interference is permitted.
Which of the statements given above is/ are correct?
a) 1 only
b) 2 and 3 only
c) 1 and 3 only
d) 1, 2 and 3

Q.9) India is a party to the Ramsar convention and has declared many areas as Ramsar sites. Which of the
following statements best describes as to how should maintain these sites in the context of this
convention?
a) Keep all the sites completely in accessible to man so that they will not be exploited
b) Conserve all the sites through ecosystem approach and permit tourism and recreation only
c) Conserve all the sites through ecosystem approaches for a period without any exploitation with
specific criteria and specific period each site, and the allow sustainable use of them by future generations
d) Conserve all the sites through ecosystems approach and allow their simultaneous sustainable use

Created with by ForumIAS.com – The Knowledge Network for Civil Services.


Visit http//forumias.com New! http://forumiasacademy – Prelims & Mains Test Series

ForumIAS
PRELIMS MARATHON COMPILATION FOR THE MONTH OF JUNE, 2018

Q.10) Consider the following pairs:


Protected areawell known for
1. Bhiterkanika, Orissa salt water
2. Deserts national park (Rajasthan) great Indian Bustard
3. Eravikulam, KerlaHoolak Gibbon
Which of the pairs given above is / are correctly matched?
a) 1 only
b) 1 and 2 only
c) 2 only
d) 1, 2 and 3

Q.11) Consider the following statements:


1. Biodiversity hotspots are located only in tropical regions.
2. India has four Biodiversity hotspots i.e., Eastern Himalyas, western Himalayas, western Ghats and
Andaman and Nicobar island
Which of the statement given above is/ are correct?
a) 1 only
b) 2 only
c) Both 1 and 2
d) Neither 1 nor 2

Q.12) Which among the following National Highway routes is the longest?
a) Agra- Mumbai
b) Chennai- Thane
c) Kokata- Hajira
d) Pune- Machilipatnam

Q.13 Consider the following


1. Mahadeo Hills
2. Sahyadri Parvat
3. Satpura Range
What is the correct sequence of the above from the north to the south?
a) 1,2,3
b) 2,1,3
c) 1,3,2
d) 2,3,1

Q.14) Lake Sambhar is nearest to which one of the following scities of Rajasthan?
a) Bharatpur
b) Jaipur
c) Jodhpur
d) Udaipur
Created with by ForumIAS.com – The Knowledge Network for Civil Services.
Visit http//forumias.com New! http://forumiasacademy – Prelims & Mains Test Series

ForumIAS
PRELIMS MARATHON COMPILATION FOR THE MONTH OF JUNE, 2018

Q.15) Excessive release of the pollutant carbon monoxide (CO) into the air may produce a condition in
which oxygen supply in the human body decreases. What cause this condition ?
a) when inhaled into the human body, CO, is converted into CO2
b) the inhaled CO has much higher affinity for hemoglobin as compared to oxygen
c) The inhaled CO destroys the chemical structure of hemoglobin
d) The inhaled CO adversely affects the respiratory centre in the brain

Q.16) Consider the following statements:


1. The taxus tree is naturally found in the Himalayas.
2. The taxustree is listed in the Red Data book
3. A Drug called “taxol” is obtained from Taxus tree is effective against Parkinson’s disease.
Which of the statements given above is / are correct?
a) 1 only
b) 1 and 2 only
c) 2 and 3 only
d) 1, 2 and 3

Q.17) A pesticide which is chlorinated hydrocarbon is sprayed on a food crop. The food chain is Food
crop- rat – snake – Hawak. In this food chain, the highest concentration of the pesticide would be
accumulate in which one of the following?
a) Food crop
b) Rat
c) snake
d) Hawk

Q.18) Three of the following criteria have contributed to the recognition of western Ghats- Sri Lanka and
Indo Burma regions are hotspots of biodiversity:
1. Species richness
2. Vegetation density
3. Endemism
4. Ethno- botanical importance
5. Threat perception
6. Adaptation of flora and fauna to warm arid humid conditions
Which three of the above are correct criteria in context?
a) 1, 2 and 6
b) 2, 4 and 6
c) 1, 3 and 5
d) 3, 4 and 6

Created with by ForumIAS.com – The Knowledge Network for Civil Services.


Visit http//forumias.com New! http://forumiasacademy – Prelims & Mains Test Series

ForumIAS
PRELIMS MARATHON COMPILATION FOR THE MONTH OF JUNE, 2018

Q.19) Human activities in the recent past have caused the increased concentration of carbon dioxide in
the atmosphere, but a lot of its does not remain in the lower atmosphere because of
1. Its Escape into the outer stratosphere
2. The photosynthesis by phytoplankton in the oceans.
3. The trapping of air in the polar ice caps
Which of the statement given above is/ are correct?
a) 1 and 2
b) 2 only
c) 2 and 3 only
d) 3 only

Q.20) If tropical rain forests is removed, it does not regenerate quickly as compared to a tropical
deciduous forests. This is because
a) The soil of rain forest is deficient in nutrient
b) propagules of the trees in a rain forest have poor viability
c) The rain forest species are slow growing
D) Exotic species invade the fertile soil of rain forest

Q.21) Where do the Sahariya tribals, who were recently in the news, live?
a) Andhra Pradesh
b) Assam
C) Rajasthan
d) Orissa

Q.22) Consider the following statements :


1. India is the only country in the world producing all the five known commercial varieties of silk.
2. India is the largest producer of sugar in the world.
Which of the statements given above is/are correct?
a) 1 only
b) 2 only
c) Both 1 and 2
d) neither 1 nor 2

Q.23) Which one of the following statements is not correct?


a) Rourkela Steel Plan, the first integrated steel plant in the Public Sector of India was set up with the
Soviet Union collaboration
b) Salem Steel Plant is a premier producer of stainless steel in India
c) Maharashtra Elektrosmelt Ltd. Is a subsidiary of the Steel Authority of India Ltd.
Visakhapatanam Steel Plant is a unit of the Rashtriya Ispat Nigam Ltd.
d) Visakhapatnam Steel Plant is a unit of the Rashtriya Ispat Nigam Ltd.

Created with by ForumIAS.com – The Knowledge Network for Civil Services.


Visit http//forumias.com New! http://forumiasacademy – Prelims & Mains Test Series

ForumIAS
PRELIMS MARATHON COMPILATION FOR THE MONTH OF JUNE, 2018

Q.24) Which one of the following statements is not correct?


a) The Western Ghats are relatively higher in their northern region
b) The Anai Mudi is the highest peak in the Western Ghats
c) Tapi river lies to the south of Satpura
d) The Narmada and the Tapi river valleys are said to be old rift valleys

Q.25) The Himalayan Range is very rich in species diversity. Which one among the following is most
appropriate reason for this phenomenon?
a) it has high rainfall that supports luxuriant vegetative growth
b) It is a confluence of different bio geographical zones
c) Exotic and invasive species have not been introduce in this region
d) It has less human interference

Q.26) there is a concern over the increase in harmful algal blooms in the seawater of India. What could be
the causative factors for this phenomenon?
1. Discharge of nutrients from the estuaries
2. Run-off from the land during the monsoon.
3. Upwelling in the seas.
Select the correct answer from the codes given below:
a) 1 only
b) 1 and 2 only
c) 2 and 3 only
d) 1, 2 and 3

Q.27) consider the following:


1. Photosynthesis
2. Respiration
3. Decay and organic matter
4. Volcanic action
Which of the above add carbon dioxide to the carbon cycle on Earth?
a) 1 and 4 only
b) 2 and 3 only
c) 2, 3 and 4 only
d) 1, 2, 3 and 4 only

Q.28) Gandhi Sagar Dam is a part of which one of the following?


a) Chambal Project
b) Kosi Project
c) Damodar Valley Project
d) Bhakra Nangal Project

Created with by ForumIAS.com – The Knowledge Network for Civil Services.


Visit http//forumias.com New! http://forumiasacademy – Prelims & Mains Test Series

ForumIAS
PRELIMS MARATHON COMPILATION FOR THE MONTH OF JUNE, 2018

Q.29) Which one of the following is the correct sequence of the given hills starting from the north and
going towards the south?
a) nallamalai Hills- Nilgiri Hills- Javadi Hills – Anaimalai Hills
b) Anaimalai Hills- Javadi Hills- nilgiri Hills anaimalai
c) Nallamalai Hills – Javdi Hills- Nilgiri Hills- Anaimalai Hills
d) Anaimalai Hills- Nilgiri Hills- Javadi Hills- Nallamalai Hills

Q.30) Consider the following statements :


1. Liquefied Natural Gas (LNG) is liquefied under extremely cold temperatures and high pressure to
facilitate storage or transportation in specially designed vessels.
2. First LNG terminal in India was built in Hassan.
3. Natural Gas Liquids (NGL) are separated from LPG and these include ethane, propane, butane and
natural gasoline.
Which of the statements given above is/are correct?
a) 1 Only
b) 1 and 3
c) 2 and 3
d) 1,2 and 3

Q.31) Which one of the following statements is not correct?


a) There is no definition of the Scheduled Tribe in the constitution of India
b) North-East India accounts for a little over half of the country’s tribal population
c) The people known as Todas live in the Nilgiri area
d) Lotha is a language spoken in Nagaland

Q.32) A sandy and saline area is the natural habitat of an Indian animal species. The animal has no
predators in that area but its existence is threatened due to destruction of its habitat. Which one of the
following could be that animals?
a) Indian wild buffalo
b) Indian wild ass
c) Indian wild boar
d) Indian gazelle

Q.33) Biodiversity forms the basis for human existence in the following ways:
1. Soil formation
2. Prevention of soil erosion
3. Recycling of waste
4. Pollination of crop
Select the correct answer using the codes given below:
a) 1, 2 and 3 only
b) 2, 3 and 4 only
c) 1 and 4 only
Created with by ForumIAS.com – The Knowledge Network for Civil Services.
Visit http//forumias.com New! http://forumiasacademy – Prelims & Mains Test Series

ForumIAS
PRELIMS MARATHON COMPILATION FOR THE MONTH OF JUNE, 2018

d) 1, 2, 3 and 4

Q.34) which one of the following is not a site for in situ method of conservation of flora?
a) Biosphere reserve
b) Botanical Garden
C) National park
D) Wild life sanctuary

Answer Key

Qs. Ans. Qs. Ans. Qs. Ans. Qs. Ans.


1 B 11 D 21 C 31 B
2 B 12 C 22 A 32 B
3 D 13 C 23 A 33 D
4 D 14 B 24 A 34 B
5 B 15 B 25 B
6 A 16 B 26 D
7 C 17 D 27 C
8 C 18 C 28 A
9 D 19 C 29 C
10 B 20 A 30 B

Explanation

1. ‘Carbon Credit’: A permit that allows the holder to emit one ton of carbon dioxide. Credits are
awarded to countries or group that have reduced their green house gases below their emission quota.
Carbon credits can be traded in the international market at their current market price. The carbon credit
system was ratified in conjunction with the Kyoto Protocol. Its goal is to stop the increase of carbon
dioxide emissions.

2. The United Nations Framework Convention on climate change (UNFCCC) is an international


environmental treaty produced at the United Nations Conference on Environment and development
(UNCED), informally known as the Earth Summit , held in Rio de Janeiro from June, 1992. The objective
of the treaty is to stabilize greenhouse gas concentration in the atmosphere at a level that would prevent
dangerous anthropogenic interference with the climate system,

Created with by ForumIAS.com – The Knowledge Network for Civil Services.


Visit http//forumias.com New! http://forumiasacademy – Prelims & Mains Test Series

ForumIAS
PRELIMS MARATHON COMPILATION FOR THE MONTH OF JUNE, 2018

3. Ambient temperature means when the temperature of the surrounding and will be the same as room
temperature. Soot mostly form diesel engines is blocking snow and ice from reflecting sunlight, which is
contributing to “ near worldwide melting of ice” and as a quarter of all observed global warming.
SF6 is the most potent greenhouse gas that has been evaluated with a global warming potential of 22,800
times that of CO2 when compared over a 100 year period.
Greenhouse effect is caused by atmosphere trace gases such as carbon dioxide (CO2), ozone (O2)and
water vapour (H2O). Infrared radiation from the earth stored temporarily in the atmosphere. Of all these
trace gases, water vapour represent the most important constituent.It contributes to the natural
greenhouse warming process by approximately 60%

4. Sustainable development is an organising principle for human life on a finite planet, it ties together the
concern for the carrying capacity of natural systems with the social and economic challenges faced by
humanity.
The term “ sustainable development” rose to significance after it was used by Brundtland Commission in
its 1978 report ‘our common future’ in the report, commission coined what has become the most after –
quoted definition of sustainable development “ development that meets the needs of the present without
compromising the ability of future generations to meet their own needs.”
The concept of sustainable development has in the past most often been broken out into three constituent
domains: environmental sustainability, economic sustainability and social sustainability.

5. Mustard, wheat are Rabi crops. Whereas Cotton, Groundunt, Maize are Kharif crops.

6. The largest Buddhist monastery in India is In Arunachal Pradesh. Tawang Monastery in Aruncachal
Pradesh is the largest Monastery in India. It was founded near the small town of tawang in the North-
Western part of Arunachal Pradesh by Merak Lama Godrej Gyatso in 1980-81

7. Damodar Valley Corporation (DVC) is the first multipurpose river valley project of independent
Indian, in 1948, on the river Damodar the DVC’s main projects include four dams at Maitho, Panchet and
Konar, with connected hydroelectric power stations, Thermal Power Station at Bokarow, Chandrapura,
Durgapura, Mejia and also one gas turbine.

8. Biosphere reserve is declared to conserve some representative ecosystems as a whole for long term in
sites. Since 1970, legislations like wildlife protections Act are used to specify borders of National park and
wild life sanctuary.
Biosphere reserve is an environment sensitive area with protected status managed primarily to preserve
biodiversity of natural ecological conditions and its sustainable use. Each Biosphere Reserve is intended
to contribute to the conservation of landscapes, ecosystems species and genetic variation.
There are three zone in a wild life sanctuary-core zone (no interference allowed), buffer zone (eco-
development and forest improvement, regulated pilgrimage, research activities are allowed) and tourism
zone (tourism is allowed)

Created with by ForumIAS.com – The Knowledge Network for Civil Services.


Visit http//forumias.com New! http://forumiasacademy – Prelims & Mains Test Series

ForumIAS
PRELIMS MARATHON COMPILATION FOR THE MONTH OF JUNE, 2018

9. Ramsar convention on wet lands – The convention on wetlands of International importance, called the
Ramsar Convention is an intergovernmental treaty that provides the frame work for national action and
international cooperation for the conservation and wise use Metlands and their resources the Ramsar
convention is the only global environment Treaty that deals with a particular ecosystem the treaty was
adopted in the Iranian city of Ramsar in 1971 and the convention’s member countries cover geographical
region of planet.
The Ramsar Mission – The convention’s mission is “conservation and wise use of all wetlands through
local national actions and international cooperation, as contribution towards achieving sustainable
development throughout the world.”
The convention uses a broad definitions of the types of Wetland covered in its mission , including lakes
and rivers , swamp and mashes , wet grassland and peatlands oases , estuaries, deltas and tidal flats, near
shore marine areas mangroves and coral reefs, and human made site such as fish ponds, rice paddies,
reservoirs and salt pans.
The wise use concept – At the center of the Ramsar philosophy is the “Wise use” concept. The wise use
of wetlands is defined as the maintenance of their ecological character, achieved through the
implementation of ecosystem approaches within the context of sustainable development”. Wise use
therefore has at its heart the conservation and sustainable use of wetlands and their resources, for the
benefit of humankind.

10. Bhitarkankika, National park is located in the kendrapara district of Odisha the park home to the
endangered saltwater crocodile, Indian python king cobra , black ibis, dartness and many other species of
flora and fauna. The endanged Indian Bustard is the major attraction and the desert national park,
Rajasthan

11. A biodiversity hotspot is biogeographical regions that is both a significant reservoir of biodiversity
and is threatened with destructions. Biodiversity hotspots are areas that support natural ecosystem that a
largely intact and where native species and communities associated with these ecosystem are well
presented. They are also areas a high diversity of locally endemic species, which all species that all not
found or are rarely outside the hotspot.
Western Himalayas and Andaman and Nicobar island are not biodiversity hotspots. Hotspots are also
located in temperate region eg. California floristic province, Japanese island.

12. Kolkata – Hajira (NH-6) – 1949 km.


Chennai – Thane (NH-4) – 1235 km.
Agra Mumbai (NH-3) – 1161 km
Pune - Machilipatnam (NH-9) – 841 km

13. Satpura range commences from Rajpipla hills in west, through Mahadeo hills, it extends Maikala
range I n the east. Sahayadris are Western Ghats running from Tapti valley to north of Kanyakumari in
North-South direction

Created with by ForumIAS.com – The Knowledge Network for Civil Services.


Visit http//forumias.com New! http://forumiasacademy – Prelims & Mains Test Series

ForumIAS
PRELIMS MARATHON COMPILATION FOR THE MONTH OF JUNE, 2018

14. Jaipur is the nearest city to the Lake Sambhar in Rajasthan Sambhar is India’s largest salt lake.

15. When carbon dioxide is inhaled the CO combines to form Carboxy hemoglobin (COHb). Carboxy
hemoglobin (COHb) is a stable complex of carbon monoxide and hemoglobin that red blood cells upon
contact with carbon monoxide (CO) large quantities of CO hinders the ability of Hb to deliver oxygen to
the body CO is produced is normal metabolism and is also a common chemical. Tobacco smoking
(through carbon monoxide inhalation) raise the blood levels of COHb by a factor of several times from its
normal concentration

16. The yew tree ( Taxusbaccata) is so poisonous that eating a handful of needles can kill a person. Yet a
chemical found in yew bark has proven to be an effective anti cancer medicine Demand for his drug,
known as a paclitaxel , or taxol became so high that in the 1990s, several species of yew trees were
threatened from over harvesting. Today chemists make the medicine from other, more abundant
chemicals in yew trees, or from cells grown in vats of liquid, spraring the trees. These methods all harness
the power of nature to grow chemicals that would be nearly impossible to make from scratch in a lab.

17. Biomagnification refers to the process by which certain substance such a pesticide or heavy metals
move up the food chain, work their way into rivers or lakes and are eaten by aquatic organisms such as
fish, which is turn are eaten by large birds, animals or humans. The substance become concentrated in
tissues or internal organs as they move up the chains. In this given food chain food crop ‘ rat ‘ snake ‘
hawk , the highest concentration of the pesticide would accummulate

18 Biodiversity hotspots are first identified as areas rich in endemic plants and containing a high diversity
of species. Their ecosystem which were suffering the threats of outsiders and with time, lost most of their
original habitat are under protection. Two specific criteria in defining the hotspots are first a specific
territory must contain a minimum of 1,500 species of vascular plants, Equalling to more then 0.5% of the
world ‘s total plant species as endemics Second, it has to value lost at least 70% of its original habitat.

19. Phytoplanktons absorb carbon dioxide and release oxygen through photosynthesis
Since CO2 is heavier than air, it sinks oceans than moving up also, during winters, it is observed that CO2
is trapped in ice caps.

20. actually the soil tropical rainforests is very poor there are several reasons for the poor soil of tropical
rainforests:
The soil is highly acidic. The roots of plants rely on an acidity difference between the roots and the soil in
order to absorb nutrients. When the soil is acidic, there is little difference and therefore little absorption of
nutrient from the soil.

Created with by ForumIAS.com – The Knowledge Network for Civil Services.


Visit http//forumias.com New! http://forumiasacademy – Prelims & Mains Test Series

ForumIAS
PRELIMS MARATHON COMPILATION FOR THE MONTH OF JUNE, 2018

The type of clay particles present in tropical rainforest soil has poor ability to trap nutrients and stop
them from washing away even if human artificially add nutrients to the soil. The nutrients mostly wash
away and are not absorbed by the plants
The high temperature and moisture of tropical rainforests causes dead organic matter in the soil
decompose more quickly than in other climates, thus releasing and loosing its nutrients rapidly
The high volume of rain in tropical rain forests washes nutrients out of soil more quickly than in other
climates.

21. Sahariya, the only primitive tribe of the Rajasthan state, resides in the Shahabad and Kishanganj
Panchayat Samiti’s of Baran District, They are mostly under privileged group. The name Sahariya is ssaid
to be derived from the Arabian world Shava or wilderness meaning residents of jungle

22. The five major tyupes of silk of commercial importance found in India are Mulberry, Tasar, Oak
Tasar. Muiga, Eri. India is second largest producer of sugar after Brazil in the world.

23. Rourkela Steel Plant, located in Sundargarh district (Odisha) was set up in collaboration with the
German Firm Krupps and Demag and was commissioned in 1959 The plant has a capacity to roll 1,86,000
tonnes of not rolled carbon and stainless steal flat product and 70,000 tonnes of cold rolled stainless steel
and coils per annum.

24. The Narmada and Tapi are two old rift valleys formed due to faulting and drained to the west joining
ultimately in Arabian Sea. The Western Ghats are relatively higher in their southern region. The highest
peak (2696m) of anaimudi is the Central point from where three ranges radiate in three directions – the
Cardamen hills to the south, the Anamalai to the north and the Palini hills to the North-east. Tapi river
lies to the south of Satpura and north of Ajanta ranges.

25. Bio geographically, the Himalayan mountain Range straddles a transition zone between the Palearctic
and indo – Malayan realms. Species from both realms are represented in the hotspot. In addition,
Geological, climatic and altitudinal variations in the hotspot, as well as topographic complexity,
contribute to the biological diversity and hence the species diversity of the mountain along their east west
and north – south axes.
India falls in the confluence of the three major biogeographic realms India- Myanmar, Eurasian Afro-
tropical and is one of the 12 mega biodiversity countries of the world. It also represents two biodiversity
hotspots of the world namely western Ghats and North Eastern Himalayas.

26. Causative factors blooms: upwelling, formation of mud banks, nutrient discharges from estuaries and
run off from the land during south west and northeast monsoon cause some algae blooms in coastal
waters. The changing patterns of nutrient ratio of the costal and the open ocean water due to
anthropogenic activities increased aquaculture operations leading to enrichment of coastal waters,

Created with by ForumIAS.com – The Knowledge Network for Civil Services.


Visit http//forumias.com New! http://forumiasacademy – Prelims & Mains Test Series

ForumIAS
PRELIMS MARATHON COMPILATION FOR THE MONTH OF JUNE, 2018

dispersal of toxic species through currents, storms, ship ballast waters and shell fish seeding activities
were some of the factors triggering the blooms.
Harmful algal blooms (HAB), lethal for human beings and marine ecosystems alike, are steadily
increasing in intensity in the Indian waters. Researchers have found out of the toxic blooms had increased
by around 15 percent over the last 12 years in Indian seas.

27. In photosynthesis the carbon dioxide is absorbs by the plants and the oxygen is released. It does not
add carbon dioxide to the carbon cycle. In respiration oxygen is converted to tissues and cells and the
oxidation products, carbon dioxide and water are given off. The most abundant gas typically released
into the atmosphere from volcanic systems is water vapour, followed by carbon dioxide and sulfur
dioxide. A composting process convert organic matters into stable compost that is odor and pathogen
free, and a poor breeding substrate for flies and other insects and releases carbon dioxide.

28. Chambal project is a joint venture of Rajasthan and Madhya Pradesh started in 1954 on Chambal river,
Rana Pratap Sasgar and Jawahar Sagar in Rajasthan and Gandhi Sagar in Madhya Pradesh are part of
Chambal project.

29. Nallamalai Hills- Javadi Hills- Nilgiri Hills- Anaimalai Hills.

30. First LNG terminal in India is Dahej (Gujrat), followed by Hazira (Gujrat) and DAbhol (Maharashtra).
Statement 1 and 3 are correct

31. North East India accounts for 12% where as Peninsular India has 88% of tribal population of India. So,
the statement 2 is incorrect. The remaining statements are correct.

32. The Indian wild ass also called khur range once extended from western India, southern Pakistan (i.e
provinces of Sindh and Baluchistan) Afghanistan, and south eastern Iran. It is unknown how to Indian
wild ass disappeared from its former haunts in parts of western India Pakistan, since the animal was
never a haunting target of Indian maharajas and colonial British officials of the British Raj. From 1958-
1960 , the wild ass became a victim of a disease known as surra, caused by Trypanosomaevansi and
transmitted by flies, which caused a dramatic decline of its population in India In November and
December 1961, the wild ass population was reduced to just 870 after to be outbreak of south American
horse sickness. Besidesdisease, the ass’s other threats include habitat degradation due to salt activities,
the invasion of the prosopisjuliflora shrub, and encroachment and grazing by the maldhari.

33. All these given factors contribute to the statement that biodiversity from the basis for human existence

34. except Botanical Gardens rest are the site for in situ method of conservation of flora. In situ
conservation of genetic resources in natural populations of plant or animal species, such as forest genetic
resource in natural population of tree species. It is the process of protecting and endangered plant or
Created with by ForumIAS.com – The Knowledge Network for Civil Services.
Visit http//forumias.com New! http://forumiasacademy – Prelims & Mains Test Series

ForumIAS
PRELIMS MARATHON COMPILATION FOR THE MONTH OF JUNE, 2018

animal species in its natural habitat, either by protecting or cleaning up the habitate itself, or by
defending the species from predators.

Created with by ForumIAS.com – The Knowledge Network for Civil Services.


Visit http//forumias.com New! http://forumiasacademy – Prelims & Mains Test Series

ForumIAS

You might also like